Arthritis

Pataasin ang iyong marka sa homework at exams ngayon gamit ang Quizwiz!

N/A TOP: Client Needs Category: Physiological Integrity (Pharmacological and Parenteral TherapiesExpected Actions/Outcomes) MSC: Integrated Process: Nursing Process (Planning) 27. A client had a total knee replacement this morning and has a continuous passive motion (CPM) machine. What activity related to the CPM does the RN delegate to the unlicensed assistive personnel? a. Placing controls out of the reach of confused clients b. Assessing the clients response to the CPM c. Teaching the clients family the rationale for the CPM d. Assessing neurovascular status of the leg in the CPM

A All activities are appropriate for the client with a CPM, but the nurse can delegate only the task of keeping controls out of reach of the confused client. All other activities would need to be performed by the RN. DIF: Cognitive Level: Application/Applying or higher

1568 OBJ: Special Questions: Prioritization TOP: Nursing Process: Planning MSC: NCLEX: Physiological Integrity 40. Which action will the nurse include in the plan of care for a 40-year-old with newly diagnosed ankylosing spondylitis? a. Advise the patient to sleep on the back with a flat pillow. b. Emphasize that application of heat may worsen symptoms. c. Schedule annual laboratory assessment for the HLA-B27 antigen. d. Assist patient to choose physical activities that allow the spine to flex.

A Because ankylosing spondylitis results in flexion deformity of the spine, postures that extend the spine (such as sleeping on the back and with a flat pillow) are recommended. HLA-B27 antigen levels are used for initial diagnosis, but are not needed annually. To counteract the development of flexion deformities, the patient should choose activities that extend the spine, such as swimming. Heat application is used to decrease localized pain. DIF: Cognitive Level: Understand (comprehension)

1576 OBJ: Special Questions: Prioritization TOP: Nursing Process: Assessment MSC: NCLEX: Physiological Integrity 44. A 28-year-old with psoriatic arthritis and back pain is receiving etanercept (Enbrel). Which finding is most important for the nurse to report to the health care provider? a. Crackles are heard in both lung bases. b. Red, scaly patches are noted on the arms. c. Hemoglobin level is 11.1g/dL and hematocrit is 35%. d. Patient reports continued back pain after a week of etanercept therapy.

A Because heart failure is a possible adverse effect of etanercept, the medication may need to be discontinued. The other information will also be reported to the health care provider but does not indicate a need for a change in treatment. Red, scaly patches of skin and mild anemia are commonly seen with psoriatic arthritis. Treatment with biologic therapies requires time to improve symptoms. DIF: Cognitive Level: Apply (application)

N/A TOP: Client Needs Category: Health Promotion and Maintenance (Self-Care) MSC: Integrated Process: Teaching/Learning 33. The nurse is caring for a client who has dysphagia caused by systemic sclerosis. What is the best intervention for the nurse to implement for this client? a. Encourage frequent, high-protein, easy to swallow foods. b. Teach the client to lie flat after meals to prevent reflux. c. Thicken liquids to a nectar or honey consistency. d. Have the client hyperextend his or her neck while swallowing.

A Clients with dysphagia frequently have esophageal motility problems, and swallowing becomes difficult. This, combined with malabsorption, leads to a malnourished client. Frequent small meals consisting of high-protein and easy to swallow foods are best. Clients should eat only in an upright position to reduce choking. Thickening liquids may help, but this does not address the malnutrition. Hyperextending the neck may help, but specific techniques should be determined by a swallowing study. DIF: Cognitive Level: Application/Applying or higher

N/A TOP: Client Needs Category: Physiological Integrity (Physiological AdaptationPathophysiology) MSC: Integrated Process: Nursing Process (Assessment) 37. A client with rheumatoid arthritis had abdominal surgery and has returned to the postoperative nursing unit. The client is unable to use the incentive spirometer correctly, demonstrating limited lung volume and fatiguing easily. What action by the nurse takes priority? a. Notify the physician immediately. b. Have respiratory therapy re-instruct the client. c. Assess for pain and medicate if necessary. d. Let the client rest for a few hours.

A Clients with rheumatoid arthritis can have cervical spine involvement resulting in subluxation. This may lead to decreased respiratory function and can be life threatening. This client was recently intubated for an operation and so is at higher risk for this problem. The nurse should notify the physician immediately and continue assessing the client. DIF: Cognitive Level: Application/Applying or higher

1581 TOP: Nursing Process: Planning MSC: NCLEX: Physiological Integrity 18. The nurse determines that colchicine has been effective for a patient with an acute attack of gout upon finding a. relief of joint pain. b. increased urine output. c. elevated serum uric acid. d. increased white blood cells (WBC).

A Colchicine produces pain relief in 24 to 48 hours by decreasing inflammation. The recommended increase in fluid intake of 2 to 3 L/day would increase urine output but would not indicate the effectiveness of colchicine. Elevated uric acid levels would result in increased symptoms. The WBC count might decrease with decreased inflammation, but would not increase. DIF: Cognitive Level: Understand (comprehension)

N/A TOP: Client Needs Category: Health Promotion and Maintenance (Self-Care) MSC: Integrated Process: Teaching/Learning 4. An older adult client is scheduled for knee replacement surgery. Which statement by the client indicates a need for further preoperative instruction? a. I need to keep my leg positioned away from my body. b. I may have a continuous passive motion machine for a few days. c. I may need more pain medicine than I did with my hip replacement. d. I probably can get back to work within 2 to 3 weeks.

A Dislocation is not a problem with knee replacement surgery, so the client does not need to keep his or her leg abducted. The other statements indicate accurate understanding of the instructions. DIF: Cognitive Level: Application/Applying or higher

N/A TOP: Client Needs Category: Physiological Integrity (Basic Care and ComfortNon-Pharmacological Comfort Interventions) MSC: Integrated Process: Teaching/Learning 18. A client with a history of rheumatoid arthritis will be starting drug therapy with etanercept (Enbrel). What is most important for the nurse to teach the client? a. The correct technique for subcutaneous injections b. How to self-monitor blood glucose levels c. How to set up and prime the IV tubing d. How to calculate the dosage based on symptoms

A Enbrel is a parenteral medication that is given by subcutaneous injection. The client and/or the family will need to be taught how to give a subcutaneous injection correctly. Blood glucose levels should not be affected by this medication. The medication is not administered IV. Drug dosages are not changed and recalculated by the client. DIF: Cognitive Level: Application/Applying or higher

307 KEY: Autoimmune disorders| laboratory values MSC: Integrated Process: Nursing Process: Assessment NOT: Client Needs Category: Physiological Integrity: Reduction of Risk Potential 1. Which finding will the nurse expect when assessing a 58-year-old patient who has osteoarthritis (OA) of the knee? a. Discomfort with joint movement b. Heberdens and Bouchards nodes c. Redness and swelling of the knee joint d. Stiffness that increases with movement

A Initial symptoms of OA include pain with joint movement. Heberdens nodules occur on the fingers. Redness of the joint is more strongly associated with rheumatoid arthritis (RA). Stiffness in OA is worse right after the patient rests and decreases with joint movement. DIF: Cognitive Level: Understand (comprehension)

N/A TOP: Client Needs Category: Physiological Integrity (Reduction of Risk PotentialPotential for Alterations in Body Systems) MSC: Integrated Process: Nursing Process (Assessment) 36. The nurse is caring for a female client who has a history of chronic fatigue syndrome. Which finding is the nurse surprised to see in the clients record? a. Hemoglobin, 7.2 g/dL b. Serum creatinine, 0.9 mg/dL c. Multiple tender lymph nodes d. Newly red, swollen, warm knee

A Laboratory values are typically normal for chronic fatigue syndrome, and no laboratory test can confirm the disease. A hemoglobin level of 7.2 is very low and is not seen with chronic fatigue syndrome. The creatinine value is normal. Tender lymph nodes and inflamed joints are normal findings in chronic fatigue syndrome. DIF: Cognitive Level: Application/Applying or higher

p. 328 TOP: Client Needs Category: Health Promotion and Maintenance (Principles of Teaching/Learning) MSC: Integrated Process: Teaching/Learning 15. The nurse is caring for a client who has had hip replacement surgery 2 days before. The client reports severe pain at the surgical site despite having received 2 Vicodin (acetaminophen and hydrocodone) tablets 2 hours previously. The client is requesting IV pain medication. What is the nurses primary intervention? a. Assess the surgical site for signs of infection. b. Administer 2 more Vicodin tablets. c. Apply a large ice bag to the operative site. d. Reassure the client that the Vicodin will work soon.

A Most clients do not need IV pain medication after the first day. If the client seems to be having unusual pain, the nurse should first assess the client for other problems, such as a joint infection. If findings are normal, applying ice to the hip will help to reduce swelling and pain. It is not time for another dose of Vicodin, because it has only been 2 hours. The nurse would not contact the surgeon unless all pain methods tried did not work. The Vicodin should have worked within 2 hours, so the nurse should not tell the client that the Vicodin will work shortly. DIF: Cognitive Level: Application/Applying or higher

1. The nurse is teaching a community health class about health promotion techniques. Which statement by a student indicates a strategy to help prevent the development of osteoarthritis? a. I will keep my BMI under 24. b. I will switch to low-tar cigarettes. c. I will start jogging twice a week. d. I will have a family tree done.

A Obesity increases the stress on weight-bearing joints and contributes to the development of degenerative joint disease. Smoking does not decrease risk for osteoarthritis. Jogging increases the risk because of increased wear and tear on the joints. There is a genetic link to osteoarthritis; creating a family tree might help the client discover if there is any familial link but will not help prevent the disorder. DIF: Cognitive Level: Application/Applying or higher

1582 OBJ: Special Questions: Prioritization TOP: Nursing Process: Assessment MSC: NCLEX: Physiological Integrity 33. Which finding for a patient who is taking hydroxychloroquine (Plaquenil) to treat rheumatoid arthritis is likely to be an adverse effect of the medication? a. Blurred vision b. Joint tenderness c. Abdominal cramping d. Elevated blood pressure

A Plaquenil can cause retinopathy. The medication should be stopped. The other findings are not related to the medication although they will also be reported. DIF: Cognitive Level: Apply (application)

N/A TOP: Client Needs Category: Physiological Integrity (Reduction of Risk PotentialLaboratory Values) MSC: Integrated Process: Nursing Process (Assessment) 9. The nurse is caring for a client who is 1 day post total hip replacement. The nurse is instructing the client about how to perform quadriceps-setting exercises correctly. Which direction does the nurse provide to the client? a. Straighten your legs and push the back of your knees into the mattress. b. Straighten your legs and bring each leg separately off the mattress 6 inches. c. Raise each leg 10 inches off the bed, keep it straight, and make ankle circles. d. Bend each knee, and rapidly point your toes downward and then upward.

A Quadriceps-setting exercises are done by straightening the leg as much as possible by attempting to push the back of the knees into the mattress. The other exercises may be performed by the client as tolerated, but these items do not describe quadriceps-setting exercises. DIF: Cognitive Level: Comprehension/Understanding

1574-1575 TOP: Nursing Process: Implementation MSC: NCLEX: Physiological Integrity 11. The nurse suggests that a patient recently diagnosed with rheumatoid arthritis (RA) plan to start each day with a. a warm bath followed by a short rest. b. a short routine of isometric exercises. c. active range-of-motion (ROM) exercises. d. stretching exercises to relieve joint stiffness.

A Taking a warm shower or bath is recommended to relieve joint stiffness, which is worse in the morning. Isometric exercises would place stress on joints and would not be recommended. Stretching and ROM should be done later in the day, when joint stiffness is decreased. DIF: Cognitive Level: Apply (application)

1573 TOP: Nursing Process: Implementation MSC: NCLEX: Physiological Integrity 13. A 37-year-old patient with 2 school-age children who has recently been diagnosed with rheumatoid arthritis (RA) tells the nurse that home life is very stressful. Which response by the nurse is most appropriate? a. Tell me more about situations that are causing you stress. b. You need to see a family therapist for some help with stress. c. Your family should understand the impact of your rheumatoid arthritis. d. Perhaps it would be helpful for your family to be involved in a support group.

A The initial action by the nurse should be further assessment. The other three responses might be appropriate based on the information the nurse obtains with further assessment. DIF: Cognitive Level: Apply (application)

1573 TOP: Nursing Process: Evaluation MSC: NCLEX: Physiological Integrity 36. A patient who had arthroscopic surgery of the right knee 7 days ago is admitted with a red, swollen, and hot knee. Which assessment finding by the nurse should be reported to the health care provider immediately? a. The blood pressure is 86/50 mm Hg. b. The white blood cell count is 11,500/L. c. The patient is taking ibuprofen (Motrin). d. The patient says the knee pain is severe.

A The low blood pressure suggests that the patient may be developing septicemia as a complication of septic arthritis. Immediate blood cultures and initiation of antibiotic therapy are indicated. The other information is typical of septic arthritis and should also be reported to the health care provider, but it does not indicate any immediately life-threatening problems. DIF: Cognitive Level: Apply (application)

Chart 20-15, p. 353 TOP: Client Needs Category: Physiological Integrity (Physiological AdaptationPathophysiology) MSC: Integrated Process: Nursing Process (Assessment) 21. The school nurse removes a tick embedded in a students scalp by the hairline. Which follow-up instruction is the nurse sure to provide to the mother? a. Call your pediatrician right away if a fever or a red rash develops at the bite. b. If your child does not have symptoms within 2 weeks, you can relax. c. Call your pediatrician tomorrow to get antibiotics to prevent Lyme disease. d. Keep the site clean, but you dont have to worry about further problems.

A The mother should be instructed to monitor for early symptoms of Lyme disease (fever, rash at the site, other flulike symptoms) following a tick bite. Symptoms can appear for up to 30 days after the bite. Antibiotics are not prescribed as a preventive measure. Because Lyme disease can cause serious complications, the mother needs to monitor the childs condition carefully. DIF: Cognitive Level: Application/Applying or higher

N/A TOP: Client Needs Category: Safe and Effective Care Environment (Management of CareDelegation) MSC: Integrated Process: Communication and Documentation 28. A client with chronic gout takes probenecid (Benemid) and comes to the clinic reporting frequent severe headaches and a new gout flare. The client is frustrated because the gout had been under good control. Which question by the nurse is most helpful? a. What do you take for your headaches? b. Do you know what triggers your gout? c. Have you been following your diet? d. Did you switch from wine to beer lately?

A The nurse needs to assess what has changed for this client. The new onset of headaches should prompt the nurse to question the client about pain medications because aspirin inactivates probenecid. Gout can have triggers, but the client probably knows them by now if it has been well controlled. Nutritional therapy for gout remains controversial. Excessive alcohol can trigger an episode, but beer does not contribute any more than wine. DIF: Cognitive Level: Application/Applying or higher

1579 OBJ: Special Questions: Prioritization TOP: Nursing Process: Assessment MSC: NCLEX: Physiological Integrity 37. A 63-year-old patient hospitalized with polymyositis has joint pain, an erythematosus facial rash, eyelid edema, and a weak, hoarse voice. The priority nursing diagnosis for the patient is a. risk for aspiration related to dysphagia. b. disturbed visual perception related to swelling. c. acute pain related to generalized inflammation. d. risk for impaired skin integrity related to scratching.

A The patients vocal weakness and hoarseness indicate weakness of the pharyngeal muscles and a high risk for aspiration. The other nursing diagnoses also are appropriate but are not as high a priority as the maintenance of the patients airway. DIF: Cognitive Level: Apply (application)

p. 325 TOP: Client Needs Category: Physiological Integrity (Reduction of Risk PotentialPotential for Complications From Surgical Procedures and Health Alterations) MSC: Integrated Process: Communication and Documentation 6. The nurse is caring for a postoperative client on the medical-surgical unit following a total left hip replacement the previous day. During the assessment, the nurse notes that the clients left leg is cool, with weak pedal pulses. What is the nurses first action? a. Assess circulatory status of the right leg. b. Notify the surgeon immediately. c. Measure leg circumference at the calf. d. Check for bilateral Homans signs.

A The symptoms may represent impaired circulation or may be normal for this client. Before the surgeon is notified, the status of the nonoperative leg should be assessed and assessment findings on both legs compared with the clients baseline. Homans sign (pain in the calf on dorsiflexion of the foot) is not always indicative of a deep vein thrombosis and should not be evaluated until other assessments are made. Measuring calf circumference would provide additional data related to deep vein thrombosis. DIF: Cognitive Level: Application/Applying or higher

N/A TOP: Client Needs Category: Health Promotion and Maintenance (Self-Care) MSC: Integrated Process: Teaching/Learning 35. The nurse is working at a clinic, where several clients are waiting to be seen. Which client does the nurse assess first? a. Client with temporal arteritis with new onset of blurry double vision b. Client with polymyalgia rheumatica with low-grade fever and fatigue c. Client with polymyositis reporting generalized rash and joint pain d. Client with ankylosing spondylitis who presents with back pain and weight loss

A Vision changes in a client with temporal arteritis are a dangerous sign that warrants immediate medical attention. The other clients signs and symptoms are commonly seen with their conditions and may be addressed on a more routine basis. DIF: Cognitive Level: Application/Applying or higher

N/A TOP: Client Needs Category: Physiological Integrity (Reduction of Risk PotentialPotential for Alterations in Body Systems) MSC: Integrated Process: Nursing Process (Implementation) 14. A client who has had total hip replacement surgery asks the nurse when she will be able to use a regular-height toilet seat again. What is the nurses best response? a. As soon as you are able to walk without a limp. b. As soon as the staples are removed from the incision. c. When you are off pain medication and warfarin (Coumadin). d. When you can hold your leg 6 inches off the bed for 5 full minutes.

A When the client is able to walk without a limp, the artificial joint is seated sturdily enough in place that it will not be dislocated or dislodged by overflexing it. At that time, the client will no longer need assistive devices or ambulatory aids. With staples removed, holding the leg off the bed and taking Coumadin do not affect readiness to bend the hip enough to use a regular toilet seat. DIF: Cognitive Level: Comprehension/Understanding

306 KEY: Rheumatoid arthritis| nursing assessment| musculoskeletal system| visual disturbances| autoimmune disorder| sensory system MSC: Integrated Process: Nursing Process: Assessment NOT: Client Needs Category: Physiological Integrity: Reduction of Risk Potential 14. The nurse is working with a client who has rheumatoid arthritis (RA). The nurse has identified the priority problem of poor body image for the client. What finding by the nurse indicates goals for this client problem are being met? a. Attends meetings of a book club b. Has a positive outlook on life c. Takes medication as directed d. Uses assistive devices to protect joints

A All of the activities are appropriate for a client with RA. Clients who have a poor body image are often reluctant to appear in public, so attending public book club meetings indicates that goals for this client problem are being met. DIF: Evaluating/Synthesis

294 KEY: Client teaching| health promotion| osteoarthritis| weight loss MSC: Integrated Process: Teaching/Learning NOT: Client Needs Category: Health Promotion and Maintenance 2. A nurse in the family clinic is teaching a client newly diagnosed with osteoarthritis (OA) about drugs used to treat the disease. For which medication does the nurse plan primary teaching? a. Acetaminophen (Tylenol) b. Cyclobenzaprine hydrochloride (Flexeril) c. Hyaluronate (Hyalgan) d. Ibuprofen (Motrin)

A All of the drugs are appropriate to treat OA. However, the first-line drug is acetaminophen. Cyclobenzaprine is a muscle relaxant given to treat muscle spasms. Hyaluronate is a synthetic joint fluid implant. Ibuprofen is a nonsteroidal anti-inflammatory drug. DIF: Remembering/Knowledge

314 KEY: Systemic lupus erythematosus| autoimmune disease| renal system MSC: Integrated Process: Nursing Process: Analysis NOT: Client Needs Category: Physiological Integrity: Reduction of Risk Potential 19. A client who has had systemic lupus erythematosus (SLE) for many years is in the clinic reporting hip pain with ambulation. Which action by the nurse is best? a. Assess medication records for steroid use. b. Facilitate a consultation with physical therapy. c. Measure the range of motion in both hips. d. Notify the health care provider immediately.

A Chronic steroid use is seen in clients with SLE and can lead to osteonecrosis (bone necrosis). The nurse should determine if the client has been taking a steroid. Physical therapy may be beneficial, but there is not enough information about the client yet. Measuring range of motion is best done by the physical therapist. Notifying the provider immediately is not warranted. DIF: Applying/Application

292 KEY: Musculoskeletal system| musculoskeletal assessment| nursing assessment| osteoarthritis MSC: Integrated Process: Nursing Process: Assessment NOT: Client Needs Category: Health Promotion and Maintenance 37. A client takes celecoxib (Celebrex) for chronic osteoarthritis in multiple joints. After a knee replacement, the health care provider has prescribed morphine sulfate for postoperative pain relief. The client also requests the celecoxib in addition to the morphine. What action by the nurse is best? a. Consult with the health care provider about administering both drugs to the client. b. Inform the client that the celecoxib will be started when he or she goes home. c. Teach the client that, since morphine is stronger, celecoxib is not needed. d. Tell the client he or she should not take both drugs at the same time.

A Despite getting an opioid analgesic for postoperative pain, the nurse should be aware that the client may be on other medications for arthritis in other joints. The nonsteroidal anti-inflammatory drug celecoxib will also help with the postoperative pain. The nurse should consult the provider about continuing the celecoxib while the client is in the hospital. The other responses are not warranted, as the client should be restarted on this medication postoperatively. DIF: Applying/Application

312 KEY: Rheumatoid arthritis| autoimmune disorder| coping| psychosocial response MSC: Integrated Process: Nursing Process: Evaluation NOT: Client Needs Category: Psychosocial Integrity 15. A client is started on etanercept (Enbrel). What teaching by the nurse is most appropriate? a. Giving subcutaneous injections b. Having a chest x-ray once a year c. Taking the medication with food d. Using heat on the injection site

A Etanercept is given as a subcutaneous injection twice a week. The nurse should teach the client how to self-administer the medication. The other options are not appropriate for etanercept. DIF: Understanding/Comprehension

302 KEY: Joint replacement| continuous passive motion machine| infection control| delegation MSC: Integrated Process: Nursing Process: Implementation NOT: Client Needs Category: Safe and Effective Care Environment: Safety and Infection Control 28. A client recently diagnosed with systemic lupus erythematosus (SLE) is in the clinic for a follow-up visit. The nurse evaluates that the client practices good self-care when the client makes which statement? a. I always wear long sleeves, pants, and a hat when outdoors. b. I try not to use cosmetics that contain any type of sunblock. c. Since I tend to sweat a lot, I use a lot of baby powder. d. Since I cant be exposed to the sun, I have been using a tanning bed.

A Good self-management of the skin in SLE includes protecting the skin from sun exposure, using sunblock, avoiding drying agents such as powder, and avoiding tanning beds. DIF: Evaluating/Synthesis

320 KEY: Gout| musculoskeletal system| patient education| nutrition MSC: Integrated Process: Teaching/Learning NOT: Client Needs Category: Physiological Integrity: Physiological Adaptation 23. A nurse is teaching a client with psoriatic arthritis about the medication golimumab (Simponi). What information is most important to include? a. Avoid large crowds or people who are ill. b. Stay upright for 1 hour after taking this drug. c. This drug may cause your hair to fall out. d. You may double the dose if pain is severe.

A This drug has a Food and Drug Administration black box warning about opportunistic or other serious infections. Teach the client to avoid large crowds and people who are ill. The other instructions do not pertain to golimumab. DIF: Applying/Application

315 KEY: Systemic lupus erythematosus| autoimmune disorders| patient education| self-care| fever MSC: Integrated Process: Teaching/Learning NOT: Client Needs Category: Physiological Integrity: Physiological Adaptation 36. A client comes to the family medicine clinic and reports joint pain and stiffness. The nurse is asked to assess the client for Heberdens nodules. What assessment technique is correct? a. Inspect the clients distal finger joints. b. Palpate the clients abdomen for tenderness. c. Palpate the clients upper body lymph nodes. d. Perform range of motion on the clients wrists.

A Herberdens nodules are seen in osteoarthritis and are bony nodules at the distal interphalangeal joints. To assess for this finding, the nurse inspects the clients distal fingertips. These nodules are not found in the abdomen, lymph nodes, or wrists. DIF: Applying/Application

317 KEY: Systemic sclerosis| autoimmune disorder| oral care| collaboration MSC: Integrated Process: Communication and Documentation NOT: Client Needs Category: Safe and Effective Care Environment: Management of Care 22. The nurse is teaching a client with gout dietary strategies to prevent exacerbations or other problems. Which statement by the nurse is most appropriate? a. Drink 1 to 2 liters of water each day. b. Have 10 to 12 ounces of juice a day. c. Liver is a good source of iron. d. Never eat hard cheeses or sardines.

A Kidney stones are common in clients with gout, so drinking plenty of water will help prevent this from occurring. Citrus juice is high in ash, which can help prevent the formation of stones, but the value of this recommendation is not clear. Clients with gout should not eat organ meats or fish with bones, such as sardines. DIF: Understanding/Comprehension

311 KEY: Rheumatoid arthritis| autoimmune disorders| ice| pain MSC: Integrated Process: Nursing Process: Implementation NOT: Client Needs Category: Physiological Integrity: Basic Care and Comfort 18. The nurse on an inpatient rheumatology unit receives a hand-off report on a client with an acute exacerbation of systemic lupus erythematosus (SLE). Which reported laboratory value requires the nurse to assess the client further? a. Creatinine: 3.9 mg/dL b. Platelet count: 210,000/mm3 c. Red blood cell count: 5.2/mm3 d. White blood cell count: 4400/mm3

A Lupus nephritis is the leading cause of death in clients with SLE. The creatinine level is very high and the nurse needs to perform further assessments related to this finding. The other laboratory values are normal. DIF: Applying/Application

321 KEY: Psoriatic arthritis| autoimmune disorders| patient education| biologic response modifiers MSC: Integrated Process: Teaching/Learning NOT: Client Needs Category: Physiological Integrity: Pharmacological and Parenteral Therapies 24. A client in the orthopedic clinic has a self-reported history of osteoarthritis. The client reports a low-grade fever that started when the weather changed and several joints started acting up, especially both hips and knees. What action by the nurse is best? a. Assess the client for the presence of subcutaneous nodules or Bakers cysts. b. Inspect the clients feet and hands for podagra and tophi on fingers and toes. c. Prepare to teach the client about an acetaminophen (Tylenol) regimen. d. Reassure the client that the problems will fade as the weather changes again.

A Osteoarthritis is not a systemic disease, nor does it present bilaterally. These are manifestations of rheumatoid arthritis. The nurse should assess for other manifestations of this disorder, including subcutaneous nodules and Bakers cysts. Podagra and tophi are seen in gout. Acetaminophen is not used for rheumatoid arthritis. Telling the client that the symptoms will fade with weather changes is not accurate. DIF: Applying/Application

296 KEY: Joint replacement| anemia| colony-stimulating factors| nursing intervention MSC: Integrated Process: Nursing Process: Implementation NOT: Client Needs Category: Physiological Integrity: Pharmacological and Parenteral Therapies 31. A client is getting out of bed into the chair for the first time after an uncemented hip replacement. What action by the nurse is most important? a. Have adequate help to transfer the client. b. Provide socks so the client can slide easier. c. Tell the client full weight bearing is allowed. d. Use a footstool to elevate the clients leg.

A The client with an uncemented hip will be on toe-touch only right after surgery. The nurse should ensure there is adequate help to transfer the client while preventing falls. Slippery socks will encourage a fall. Elevating the leg greater than 90 degrees is not allowed. DIF: Applying/Application

322 KEY: Fibromyalgia| antidepressants| pain| pharmacologic pain management MSC: Integrated Process: Teaching/Learning NOT: Client Needs Category: Physiological Integrity: Pharmacological and Parenteral Therapies 33. A client has been diagnosed with rheumatoid arthritis. The client has experienced increased fatigue and worsening physical status and is finding it difficult to maintain the role of elder in his cultural community. The elder is expected to attend social events and make community decisions. Stress seems to exacerbate the condition. What action by the nurse is best? a. Assess the clients culture more thoroughly. b. Discuss options for performing duties. c. See if the client will call a community meeting. d. Suggest the client give up the role of elder.

A The nurse needs a more thorough understanding of the clients culture, including the meaning of illness and the ramifications of the elder not being able to perform traditional duties. This must be done prior to offering any possible solutions. If the nurse does not understand the consequences of what is suggested, the client may simply be unwilling to listen or participate in problem solving. The other options may be reasonable depending on the outcome of a better cultural understanding. DIF: Applying/Application

296 KEY: Joint replacement| Surgical Care Improvement Project (SCIP)| wound infection| antibiotics MSC: Integrated Process: Nursing Process: Implementation NOT: Client Needs Category: Safe and Effective Care Environment: Safety and Infection Control 7. The nurse on the postoperative inpatient unit assesses a client after a total hip replacement. The clients surgical leg is visibly shorter than the other one and the client reports extreme pain. While a co-worker calls the surgeon, what action by the nurse is best? a. Assess neurovascular status in both legs. b. Elevate the affected leg and apply ice. c. Prepare to administer pain medication. d. Try to place the affected leg in abduction.

A This client has manifestations of hip dislocation, a critical complication of this surgery. Hip dislocation can cause neurovascular compromise. The nurse should assess neurovascular status, comparing both legs. The nurse should not try to move the extremity to elevate or abduct it. Pain medication may be administered if possible, but first the nurse should thoroughly assess the client. DIF: Applying/Application

297 KEY: Joint replacement| abduction pillow| musculoskeletal system| older adult MSC: Integrated Process: Nursing Process: Implementation NOT: Client Needs Category: Physiological Integrity: Reduction of Risk Potential 6. What action by the perioperative nursing staff is most important to prevent surgical wound infection in a client having a total joint replacement? a. Administer preoperative antibiotic as ordered. b. Assess the clients white blood cell count. c. Instruct the client to shower the night before. d. Monitor the clients temperature postoperatively.

A To prevent surgical wound infection, antibiotics are given preoperatively within an hour of surgery. Simply taking a shower will not help prevent infection unless the client is told to use special antimicrobial soap. The other options are processes to monitor for infection, not prevent it. DIF: Applying/Application

316 KEY: Systemic lupus erythematosus| autoimmune disorders| coping| psychosocial response MSC: Integrated Process: Caring NOT: Client Needs Category: Psychosocial Integrity 21. A nurse is caring for a client with systemic sclerosis. The clients facial skin is very taut, limiting the clients ability to open the mouth. After consulting with a registered dietitian for appropriate nutrition, what other consultation should the nurse facilitate? a. Dentist b. Massage therapist c. Occupational therapy d. Physical therapy

A With limited ability to open the mouth, dental hygiene may be lacking. The nurse should encourage the client to see a dentist. The other referrals are not related to the mouth. DIF: Applying/Application

301 KEY: Joint replacement| osteoarthritis| home safety| assistive devices MSC: Integrated Process: Teaching/Learning NOT: Client Needs Category: Safe and Effective Care Environment: Safety and Infection Control 11. A client with fibromyalgia is in the hospital for an unrelated issue. The client reports that sleep, which is always difficult, is even harder now. What actions by the nurse are most appropriate? (Select all that apply.) a. Allow the client uninterrupted rest time. b. Assess the clients usual bedtime routine. c. Limit environmental noise as much as possible. d. Offer a massage or warm shower at night. e. Request an order for a strong sleeping pill.

A, B, C, D Clients with fibromyalgia often have sleep disturbances, which can be exacerbated by the stress, noise, and unfamiliar environment of the hospital. Allowing uninterrupted rest time, adhering to the clients usual bedtime routine as much as possible, limiting noise and light, and offering massages or warm showers can help. The client does not need a strong sleeping pill unless all other options fail and the client requests something for sleep. At that point a mild sleeping agent can be tried. DIF: Applying/Application

315 KEY: Systemic scleroderma| autoimmune disorders| delegation| nonpharmacologic pain management MSC: Integrated Process: Communication and Documentation NOT: Client Needs Category: Safe and Effective Care Environment: Management of Care 9. A client has rheumatoid arthritis (RA) and the visiting nurse is conducting a home assessment. What options can the nurse suggest for the client to maintain independence in activities of daily living (ADLs)? (Select all that apply.) a. Grab bars to reach high items b. Long-handled bath scrub brush c. Soft rocker-recliner chair d. Toothbrush with built-up handle e. Wheelchair cushion for comfort

A, B, D Grab bars, long-handled bath brushes, and toothbrushes with built-up handles all provide modifications for daily activities, making it easier for the client with RA to complete ADLs independently. The rocker-recliner and wheelchair cushion are comfort measures but do not help increase independence. DIF: Applying/Application

308 KEY: Rheumatoid arthritis| autoimmune disease| patient education| disease-modifying antirheumatic drugs (DMARDs)| acetaminophen MSC: Integrated Process: Teaching/Learning NOT: Client Needs Category: Physiological Integrity: Pharmacological and Parenteral Therapies 3. A client has been diagnosed with fibromyalgia syndrome but does not want to take the prescribed medications. What nonpharmacologic measures can the nurse suggest to help manage this condition? (Select all that apply.) a. Acupuncture b. Stretching c. Supplements d. Tai chi e. Vigorous aerobics

A, B, D There are many nonpharmacologic means for controlling the symptoms of fibromyalgia, including acupuncture, stretching, tai chi, low-impact aerobics, swimming, biking, strengthening, massage, stress management, and hypnosis. Dietary supplements and vigorous aerobics are not recommended. DIF: Remembering/Knowledge

311 KEY: Rheumatoid arthritis| autoimmune disorders| activities of daily living| musculoskeletal system| functional ability MSC: Integrated Process: Teaching/Learning NOT: Client Needs Category: Physiological Integrity: Basic Care and Comfort 10. A home health care nurse is visiting a client discharged home after a hip replacement. The client is still on partial weight bearing and using a walker. What safety precautions can the nurse recommend to the client? (Select all that apply.) a. Buy and install an elevated toilet seat. b. Install grab bars in the shower and by the toilet. c. Step into the bathtub with the affected leg first. d. Remove all throw rugs throughout the house. e. Use a shower chair while taking a shower.

A, B, D, E Buying and installing an elevated toilet seat, installing grab bars, removing throw rugs, and using a shower chair will all promote safety for this client. The client is still on partial weight bearing, so he or she cannot step into the bathtub leading with the operative side. Stepping into a bathtub may also require the client to bend the hip more than the allowed 90 degrees. DIF: Applying/Application

304 KEY: Rheumatoid arthritis| musculoskeletal system| autoimmune disorder MSC: Integrated Process: Teaching/Learning NOT: Client Needs Category: Physiological Integrity: Physiological Adaptation 2. A nurse is teaching a female client with rheumatoid arthritis (RA) about taking methotrexate (MTX) (Rheumatrex) for disease control. What information does the nurse include? (Select all that apply.) a. Avoid acetaminophen in over-the-counter medications. b. It may take several weeks to become effective on pain. c. Pregnancy and breast-feeding are not affected by MTX. d. Stay away from large crowds and people who are ill. e. You may find that folic acid, a B vitamin, reduces side effects.

A, B, D, E MTX is a disease-modifying antirheumatic drug and is used as a first-line drug for RA. MTX can cause liver toxicity, so the client should be advised to avoid medications that contain acetaminophen. It may take 4 to 6 weeks for effectiveness. MTX can cause immunosuppression, so avoiding sick people and crowds is important. Folic acid helps reduce side effects for some people. Pregnancy and breast-feeding are contraindicated while on this drug. DIF: Applying/Application

1571 TOP: Nursing Process: Assessment MSC: NCLEX: Physiological Integrity MULTIPLE RESPONSE 1. During assessment of the patient with fibromyalgia, the nurse would expect the patient to report which of the following (select all that apply)? a. Sleep disturbances b. Multiple tender points c. Cardiac palpitations and dizziness d. Multijoint pain with inflammation and swelling e. Widespread bilateral, burning musculoskeletal pain

A, B, E These symptoms are commonly described by patients with fibromyalgia. Cardiac involvement and joint inflammation are not typical of fibromyalgia. DIF: Cognitive Level: Understand (comprehension)

297 KEY: Joint replacement| delegation| abduction pillow| unlicensed assistive personnel (UAP)| nursing assessment MSC: Integrated Process: Communication and Documentation NOT: Client Needs Category: Safe and Effective Care Environment: Management of Care 6. The nurse is working with clients who have connective tissue diseases. Which disorders are correctly paired with their manifestations? (Select all that apply.) a. Dry, scaly skin rash Systemic lupus erythematosus (SLE) b. Esophageal dysmotility Systemic sclerosis c. Excess uric acid excretion Gout d. Footdrop and paresthesias Osteoarthritis e. Vasculitis causing organ damage Rheumatoid arthritis

A, B, E A dry, scaly skin rash is the most frequent dermatologic manifestation of SLE. Systemic sclerosis can lead to esophageal motility problems. Vasculitis leads to organ damage in rheumatoid arthritis. Gout is caused by hyperuricemia; the production of uric acid exceeds the excretion capability of the kidneys. Footdrop and paresthesias occur in rheumatoid arthritis. DIF: Remembering/Knowledge

305 KEY: Rheumatoid arthritis| nursing assessment| musculoskeletal system| autoimmune disorders MSC: Integrated Process: Nursing Process: Assessment NOT: Client Needs Category: Physiological Integrity: Physiological Adaptation 5. An older client returning to the postoperative nursing unit after a hip replacement is disoriented and restless. What actions does the nurse delegate to the unlicensed assistive personnel (UAP)? (Select all that apply.) a. Apply an abduction pillow to the clients legs. b. Assess the skin under the abduction pillow straps. c. Place pillows under the heels to keep them off the bed. d. Monitor cognition to determine when the client can get up. e. Take and record vital signs per unit/facility policy.

A, C, E The UAP can apply an abduction pillow, elevate the heels on a pillow, and take/record vital signs. Assessing skin is the nurses responsibility, although if the UAP notices abnormalities, he or she should report them. Determining when the client is able to get out of bed is also a nursing responsibility. DIF: Applying/Application

N/A TOP: Client Needs Category: Health Promotion and Maintenance (Self-Care) MSC: Integrated Process: Teaching/Learning 2. After hip replacement surgery, a client receives two doses of enoxaparin (Lovenox) during the day shift. What orders does the nurse anticipate for the client? (Select all that apply.) a. Laboratory draw for platelet count b. Laboratory draw for prothrombin time (PTT) c. Laboratory draw for international normalized ratio (INR) d. Order for protamine sulfate e. Order for vitamin K

A, D Lovenox is a lowmolecular-weight heparin. Side effects can include thrombocytopenia. The antidote for all heparin products is protamine sulfate, although it will not be as effective for Lovenox as it is for unfractionated heparin. DIF: Cognitive Level: Application/Applying or higher

1571 TOP: Nursing Process: Implementation MSC: NCLEX: Physiological Integrity 6. Which action will the nurse include in the plan of care for a 33-year-old patient with a new diagnosis of rheumatoid arthritis? a. Instruct the patient to purchase a soft mattress. b. Suggest that the patient take a nap in the afternoon. c. Teach the patient to use lukewarm water when bathing. d. Suggest exercise with light weights several times daily.

B Adequate rest helps decrease the fatigue and pain that are associated with rheumatoid arthritis. Patients are taught to avoid stressing joints, to use warm baths to relieve stiffness, and to use a firm mattress. When stabilized, a therapeutic exercise program is usually developed by a physical therapist to include exercises that improve the flexibility and strength of the affected joints, and the patients overall endurance. DIF: Cognitive Level: Apply (application)

1588 TOP: Nursing Process: Assessment MSC: NCLEX: Physiological Integrity 27. The nurse determines that additional instruction is needed when a patient diagnosed with scleroderma says which of the following? a. Paraffin baths can be used to help my hands. b. I should lie down for an hour after each meal. c. Lotions will help if I rub them in for a long time. d. I should perform range-of-motion exercises daily.

B Because of the esophageal scarring, patients should sit up for 2 hours after eating. The other patient statements are correct and indicate that the teaching has been effective. DIF: Cognitive Level: Apply (application)

N/A TOP: Client Needs Category: Safe and Effective Care Environment (Management of CareCollaboration with Interdisciplinary Team) MSC: Integrated Process: Caring 41. The nurse is caring for an older adult client who will be discharged home to live with an adult daughter. The client will be given prescriptions for four new medications for rheumatoid arthritis. How does the nurse ensure that the client will be able to take the medications correctly at home? a. Monitor the client self-administering medications while in the hospital. b. Include the clients daughter when teaching the client about the medications. c. Provide the client with pamphlets and information about all the medications. d. Make a chart showing which medications the client should take at different times.

B Because the client will be living with the daughter, she should be included in the teaching plan about the medications. Providing pamphlets or charts about the medications does not ensure that the client knows how to take them correctly at home. Self-administering medications may or may not be permitted by hospital policy and might be helpful, but including the daughter would be the best option. DIF: Cognitive Level: Application/Applying or higher

N/A TOP: Client Needs Category: Physiological Integrity (Pharmacological and Parenteral TherapiesAdverse Effects/Contraindications/Interactions/Side Effects) MSC: Integrated Process: Nursing Process (Implementation) 8. A client is receiving warfarin (Coumadin) daily following total hip replacement surgery. Which laboratory value requires intervention by the nurse? a. Potassium (K+), 4.2 mEq/L b. International normalized ratio (INR), 5.1 c. Prothrombin time (PT), 13.4 seconds d. Hemoglobin (Hg), 16 g/dL

B Blood levels of Coumadin will be monitored by checking daily PT and INR (in some places, only INR). The INR is critically high. The K+ is normal and is not monitored for Coumadin therapy. The PT is used in some facilities to monitor Coumadin therapy. Hemoglobin would be important to assess because a side effect of Coumadin is bleeding, and a dropping hemoglobin level would indicate that bleeding was occurring. PT and hemoglobin are within the normal range. DIF: Cognitive Level: Application/Applying or higher

N/A TOP: Client Needs Category: Physiological Integrity (Reduction of Risk PotentialPotential for Complications From Surgical Procedures and Health Alterations) MSC: Integrated Process: Nursing Process (Assessment) 13. A client had a total knee replacement earlier in the day and has a continuous femoral nerve blockade (CFNB). When entering the room to assess the client, the nurse notes that the television volume is quite loud. The client explains that it is hard to hear with all the ringing in my ears. What action by the nurse takes priority? a. Perform a neurovascular assessment on the operative extremity. b. Call another nurse to notify the anesthesiologist immediately. c. Take a full set of vital signs and discontinue the CFNB. d. Pad the siderails and instituting other seizure precautions.

B CFNB can enter the systemic circulation, causing tinnitus, nervousness, slurred speech, bradycardia, hypotension, bradypnea, and seizures. Because the client is exhibiting signs the CFNB has entered his or her circulation, the client is at risk for seizures and critical alterations in vital signs. The nurse should stay with the client and should continue to assess him or her while another nurse notifies the surgeon or the anesthesiologist. DIF: Cognitive Level: Application/Applying or higher

N/A TOP: Client Needs Category: Physiological Integrity (Reduction of Risk PotentialPotential for Complications From Surgical Procedures and Health Alterations) MSC: Integrated Process: Nursing Process (Assessment) 7. A client is admitted for a total hip replacement. Past medical history includes diabetes mellitus type 2, a heart attack 5 years ago, and allergies to sulfa drugs. The client currently takes insulin on a sliding scale and celecoxib (Celebrex). Before administering the clients medications, which action by the nurse is most appropriate? a. Take the clients blood pressure in both arms. b. Call the physician to clarify the orders. c. Schedule a preoperative electrocardiogram. d. Review the clients laboratory values.

B Celebrex is a cyclooxygenase (COX)-2 inhibitor. These drugs are thought to cause serious adverse reactions such as myocardial infarction and renal problems. This client already has coronary artery disease and a past myocardial infarction, so the nurse should discuss the order with the physician before giving the medication. Reviewing laboratory results could indicate renal impairment, but taking the clients blood pressure and scheduling an electrocardiogram (ECG) would not take priority over discussion with the physician. DIF: Cognitive Level: Application/Applying or higher

1590 TOP: Nursing Process: Implementation MSC: NCLEX: Physiological Integrity 8. Which information will the nurse include when preparing teaching materials for patients with exacerbations of rheumatoid arthritis? a. Affected joints should not be exercised when pain is present. b. Application of cold packs before exercise may decrease joint pain. c. Exercises should be performed passively by someone other than the patient. d. Walking may substitute for range-of-motion (ROM) exercises on some days.

B Cold application is helpful in reducing pain during periods of exacerbation of RA. Because the joint pain is chronic, patients are instructed to exercise even when joints are painful. ROM exercises are intended to strengthen joints and improve flexibility, so passive ROM alone is not sufficient. Recreational exercise is encouraged but is not a replacement for ROM exercises. DIF: Cognitive Level: Apply (application)

1589 OBJ: Special Questions: Prioritization TOP: Nursing Process: Diagnosis MSC: NCLEX: Physiological Integrity 38. A 46-year-old male patient with dermatomyositis is receiving long-term prednisone (Deltasone) therapy. Which assessment finding by the nurse is most important to report to the health care provider? a. The blood glucose is 112 mg/dL. b. The patient has painful hematuria. c. Acne is noted on the patients face. d. The patient has an increased appetite.

B Corticosteroid use is associated with an increased risk for infection, so the nurse should report the urinary tract symptoms immediately to the health care provider. The increase in blood glucose, increased appetite, and acne are also adverse effects of corticosteroid use but do not need diagnosis and treatment as rapidly as the probable urinary tract infection. DIF: Cognitive Level: Apply (application)

1573 TOP: Nursing Process: Implementation MSC: NCLEX: Physiological Integrity 29. Which assessment information obtained by the nurse indicates that a patient with an exacerbation of rheumatoid arthritis (RA) is experiencing a side effect of prednisone (Deltasone)? a. The patient has joint pain and stiffness. b. The patients blood glucose is 165 mg/dL. c. The patient has experienced a recent 5-pound weight loss. d. The patients erythrocyte sedimentation rate (ESR) has increased.

B Corticosteroids have the potential to cause diabetes mellitus. The finding of an elevated blood glucose reflects this side effect of prednisone. Corticosteroids increase appetite and lead to weight gain. An elevated ESR and no improvement in symptoms would indicate that the prednisone was not effective but would not be side effects of the medication. DIF: Cognitive Level: Apply (application)

1562 TOP: Nursing Process: Assessment MSC: NCLEX: Physiological Integrity 2. Which assessment finding about a patient who has been using naproxen (Naprosyn) for 6 weeks to treat osteoarthritis is most important for the nurse to report to the health care provider? a. The patient has gained 3 pounds. b. The patient has dark-colored stools. c. The patients pain has become more severe. d. The patient is using capsaicin cream (Zostrix).

B Dark-colored stools may indicate that the patient is experiencing gastrointestinal bleeding caused by the naproxen. The information about the patients ongoing pain and weight gain also will be reported and may indicate a need for a different treatment and/or counseling about avoiding weight gain, but these are not as large a concern as the possibility of gastrointestinal bleeding. Use of capsaicin cream with oral medications is appropriate. DIF: Cognitive Level: Apply (application)

1574-1575 TOP: Nursing Process: Implementation MSC: NCLEX: Psychosocial Integrity 14. Which information will the nurse include when teaching a 38-year-old male patient with newly diagnosed ankylosing spondylitis (AS) about the management of the condition? a. Exercise by taking long walks. b. Do daily deep-breathing exercises. c. Sleep on the side with hips flexed. d. Take frequent naps during the day.

B Deep-breathing exercises are used to decrease the risk for pulmonary complications that may occur with the reduced chest expansion that can occur with ankylosing spondylitis (AS). Patients should sleep on the back and avoid flexed positions. Prolonged standing and walking should be avoided. There is no need for frequent naps. DIF: Cognitive Level: Apply (application)

1581 TOP: Nursing Process: Planning MSC: NCLEX: Physiological Integrity 41. After the nurse has taught a 28-year-old with fibromyalgia, which statement by the patient indicates a good understanding of effective self-management? a. I am going to join a soccer team to get more exercise. b. I will need to stop drinking so much coffee and soda. c. I will call the doctor every time my symptoms get worse. d. I should avoid using over-the-counter medications for pain.

B Dietitians frequently suggest that patients with fibromyalgia limit their intake of caffeine and sugar because these substances are muscle irritants. Mild exercise such as walking is recommended for patients with fibromyalgia, but vigorous exercise is likely to make symptoms worse. Because symptoms may fluctuate from day to day, the patient should be able to adapt the regimen independently, rather than calling the provider whenever symptoms get worse. Over-the-counter medications such as ibuprofen and acetaminophen are frequently used for symptom management. DIF: Cognitive Level: Apply (application)

N/A TOP: Client Needs Category: Physiological Integrity (Basic Care and ComfortNutrition and Oral Hydration) MSC: Integrated Process: Nursing Process (Implementation) 34. The nurse is instructing a client about the management of systemic sclerosis. Which statement indicates that the client requires additional teaching? a. I will let my doctor know right away if I develop a fever. b. Ice packs will help relieve the aching pain in my hips and knees. c. I will wear mittens when I am in the freezer section of the grocery store. d. I will apply a rich moisturizer to my skin every morning after my shower.

B Ice packs should not be used by clients with systemic sclerosis because the cold can trigger symptoms of Raynauds phenomenon. The client should wear mittens whenever his or her hands are exposed to cold temperatures, and moisturizer should be applied daily. The client should notify the doctor if a fever develops. DIF: Cognitive Level: Application/Applying or higher

1584 TOP: Nursing Process: Implementation MSC: NCLEX: Physiological Integrity 26. A 40-year-old African American patient has scleroderma manifested by CREST (calcinosis, Raynauds phenomenon, esophageal dysfunction, sclerodactyly, and telangiectasia) syndrome. Which action will the nurse include in the plan of care? a. Avoid use of capsaicin cream on hands. b. Keep environment warm and draft free. c. Obtain capillary blood glucose before meals. d. Assist to bathroom every 2 hours while awake.

B Keeping the room warm will decrease the incidence of Raynauds phenomenon, one aspect of the CREST syndrome. Capsaicin cream may be used to improve circulation and decrease pain. There is no need to obtain blood glucose levels or to assist the patient to the bathroom every 2 hours. DIF: Cognitive Level: Apply (application)

1578 TOP: Nursing Process: Planning MSC: NCLEX: Physiological Integrity 25. The health care provider has prescribed the following collaborative interventions for a patient who is taking azathioprine (Imuran) for systemic lupus erythematosus. Which order will the nurse question? a. Draw anti-DNA blood titer. b. Administer varicella vaccine. c. Naproxen (Aleve) 200 mg BID. d. Famotidine (Pepcid) 20 mg daily.

B Live virus vaccines, such as varicella, are contraindicated in a patient taking immunosuppressive drugs. The other orders are appropriate for the patient. DIF: Cognitive Level: Apply (application)

N/A TOP: Client Needs Category: Health Promotion and Maintenance (Self-Care) MSC: Integrated Process: Teaching/Learning 3. The nurse is working with a client who will be taking 20 mg of prednisone daily for rheumatoid arthritis. Which precautions does the nurse give the client about taking this medication? a. Take this medication at bedtime because it will make you sleepy. b. Take calcium and vitamin D supplements daily. c. Eat a high-fiber diet with lots of lean meats. d. Wash your face twice a day with an antibacterial soap.

B Long-term steroid use is associated with many complications, including diabetes, infection, and osteoporosis, among others. The client should be instructed to take calcium and vitamin D supplements to help prevent osteoporosis. Prednisone does not cause constipation, so increased fiber would not be helpful. Prednisone should be taken in the morning because it may interfere with sleep if taken at bedtime. Washing the face with antibacterial soap may cause skin dryness and breakdown. DIF: Cognitive Level: Application/Applying or higher

1577-1578 TOP: Nursing Process: Evaluation MSC: NCLEX: Physiological Integrity 19. A patient with gout has a new prescription for losartan (Cozaar) to control the condition. The nurse will plan to monitor a. blood glucose. b. blood pressure. c. erythrocyte count. d. lymphocyte count.

B Losartan, an angiotensin II receptor antagonist, will lower blood pressure. It does not affect blood glucose, red blood cell (RBC) count, or lymphocytes. DIF: Cognitive Level: Apply (application)

N/A TOP: Client Needs Category: Health Promotion and Maintenance (Self-Care) MSC: Integrated Process: Teaching/Learning 11. A client who has had bilateral total knee replacements is prescribed enoxaparin sodium (Lovenox) injections twice daily for the next 3 weeks. The client asks the nurse why she has to have the medication. What is the nurses best response? a. To prevent swelling within your new knee joints. b. To prevent the formation of blood clots in your legs. c. To prevent arthritis from developing in your new knee joints. d. To prevent an infection from developing in your new knee joints.

B Lovenox is an anticoagulant that will help prevent formation of postoperative deep vein thrombosis (DVT). Lovenox does not decrease or prevent swelling, it does not prevent arthritis, and it is not an antibiotic. DIF: Cognitive Level: Comprehension/Understanding

N/A TOP: Client Needs Category: Physiological Integrity (Physiological AdaptationPathophysiology) MSC: Integrated Process: Nursing Process (Assessment) 24. The nurse is working in a primary care clinic and sees a young male client. The client is athletic and is well over 6 feet tall, with size 14 shoes. What diagnostic test does the nurse facilitate for the client? a. Coagulation studies b. Echocardiography c. Electromyelography d. Genetic testing

B Marfan syndrome is seen in athletic clients who are very tall and have large hands and feet. Echocardiography should be done for clients who may have Marfan syndrome to monitor for mitral valve prolapse and aortic aneurysm. Marfan disease has a genetic component, and genetic testing may be done, but the priority is monitoring the client for cardiac complications. DIF: Cognitive Level: Application/Applying or higher

1566 TOP: Nursing Process: Evaluation MSC: NCLEX: Physiological Integrity 34. A 31-year-old woman is taking methotrexate (Rheumatrex) to treat rheumatoid arthritis. Which information from the patients health history is important for the nurse to report to the health care provider about the methotrexate? a. The patient had a history of infectious mononucleosis as a teenager. b. The patient is trying to get pregnant before her disease becomes more severe. c. The patient has a family history of age-related macular degeneration of the retina. d. The patient has been using large doses of vitamins and health foods to treat the RA.

B Methotrexate is teratogenic, and the patient should be taking contraceptives during methotrexate therapy. The other information will not impact the choice of methotrexate as therapy. DIF: Cognitive Level: Apply (application)

1573 OBJ: Special Questions: Prioritization TOP: Nursing Process: Assessment MSC: NCLEX: Physiological Integrity 45. Which nursing action can the registered nurse (RN) delegate to unlicensed assistive personnel (UAP) who are assisting with the care of a patient with scleroderma? a. Monitor for difficulty in breathing. b. Document the patients oral intake. c. Check finger strength and movement. d. Apply capsaicin (Zostrix) cream to hands.

B Monitoring and documenting patients oral intake is included in UAP education and scope of practice. Assessments for changes in physical status and administration of medications require more education and scope of practice, and should be done by licensed nurses. DIF: Cognitive Level: Apply (application)

1565 TOP: Nursing Process: Assessment MSC: NCLEX: Physiological Integrity 3. After the nurse has finished teaching a 68-year-old patient with osteoarthritis (OA) of the right hip about how to manage the OA, which patient statement indicates a need for more teaching? a. I can take glucosamine to help decrease my knee pain. b. I will take 1 g of acetaminophen (Tylenol) every 4 hours. c. I will take a shower in the morning to help relieve stiffness. d. I can use a cane to decrease the pressure and pain in my hip.

B No more than 4 g of acetaminophen should be taken daily to avoid liver damage. The other patient statements are correct and indicate good understanding of OA management. DIF: Cognitive Level: Apply (application)

N/A TOP: Client Needs Category: Physiological Integrity (Pharmacological and Parenteral TherapiesExpected Actions/Outcomes) MSC: Integrated Process: Teaching/Learning 30. A female client with rheumatoid arthritis has taken Rheumatrex (methotrexate) for the past year to control her symptoms. The client comes to the clinic and tells the nurse that a home pregnancy test was positive. What is the nurses best response? a. You need to schedule a prenatal appointment with your obstetrician right away. b. Stop taking Rheumatrex immediately. Ill tell the physician you are pregnant. c. Continue taking the Rheumatrex, and increase the dose if you have a flare. d. See a genetic counselor to determine whether your baby will have rheumatoid arthritis.

B Rheumatrex is highly teratogenic and should not be taken during pregnancy. A prenatal appointment should be made right away, but the first priority is to stop taking methotrexate. Genetic counseling is not appropriate because the counselor will not be able to determine whether the baby will develop rheumatoid arthritis. DIF: Cognitive Level: Application/Applying or higher

1576 TOP: Nursing Process: Implementation MSC: NCLEX: Physiological Integrity 21. Which statement by a patient with systemic lupus erythematosus (SLE) indicates that the patient has understood the nurses teaching about the condition? a. I will exercise even if I am tired. b. I will use sunscreen when I am outside. c. I should take birth control pills to keep from getting pregnant. d. I should avoid aspirin or nonsteroidal antiinflammatory drugs.

B Severe skin reactions can occur in patients with SLE who are exposed to the sun. Patients should avoid fatigue by balancing exercise with rest periods as needed. Oral contraceptives can exacerbate lupus. Aspirin and nonsteroidal antiinflammatory drugs are used to treat the musculoskeletal manifestations of SLE. DIF: Cognitive Level: Apply (application)

N/A TOP: Client Needs Category: Physiological Integrity (Reduction of Risk PotentialPotential for Complications from Surgical Procedures and Health Alterations) MSC: Integrated Process: Nursing Process (Assessment) 25. The nurse is working in a clinic when a young male client presents with reports of pain with urination. The client wants testing for sexually transmitted diseases (STDs). The nurse notes that the clients eyes are red and inflamed. What question by the nurse is most important? a. Do you have more than one sexual partner? b. Do you have any new joint pain? c. What eyedrops have you used for your red eyes? d. Are you allergic to any antibiotics?

B The client has two symptoms of Reiters syndrome (urethritis and conjunctivitis). The nurse should ask about joint pain because this is the third classic manifestation of this disease. All other questions are appropriate, but before treatment is started, the client needs an accurate diagnosis. DIF: Cognitive Level: Application/Applying or higher

N/A TOP: Client Needs Category: Physiological Integrity (Physiological IntegrityPathophysiology) MSC: Integrated Process: Nursing Process (Planning) 26. A client presents with painful, inflamed fingers with small, hard, yellow nodules that have a sandy yellow drainage. Which medication does the nurse prepare to administer to the client? a. Colchicine (Colasalide) b. Allopurinol (Zyloprim) c. Methotrexate (Rheumatrex) d. Aspirin

B The client is presenting with symptoms of chronic gout, and allopurinol would be the drug of choice to reduce uric acid levels. Colchicine is used to treat acute gout attacks. Methotrexate and aspirin are not used to treat chronic gout. DIF: Cognitive Level: Application/Applying or higher

N/A TOP: Client Needs Category: Health Promotion and Maintenance (Self-Care) MSC: Integrated Process: Nursing Process (Assessment) 40. The nurse is working with a client who has severe rheumatoid arthritis in her hands. The client states that she is frustrated at mealtime because it is difficult for her to manage cups and silverware. What is the nurses best response? a. Ill have the nursing assistants set up your meal trays while you are in the hospital. b. Lets see if the occupational therapist can provide you with some utensils that are easier for you to use. c. Ill arrange for a home nursing assistant to help you with your meals after you are discharged from the hospital. d. Lets see if the physical therapist can suggest some muscle strengthening exercises for you.

B The client wishes to be more independent at mealtimes; adaptive eating utensils from the occupational therapist will help her meet this goal. Muscle-strengthening exercises will not be as effective for the clients mealtime needs. The client wishes to remain as independent as possible, so a home nursing assistant should not be suggested. DIF: Cognitive Level: Application/Applying or higher

N/A TOP: Client Needs Category: Psychosocial Integrity (Coping Mechanisms) MSC: Integrated Process: Caring 43. The nurse is caring for an older adult client who has had a hip replacement 2 days previously. Which assessment finding is the best indicator that the client does not need pain medication at this time? a. The client received 2 pain pills 2 hours ago. b. The client states that she has no pain. c. The client is sleeping quietly. d. The clients vital signs are stable.

B The clients report of pain is the best indicator of pain level, rather than vital signs, sleeping, or time of last pain medication. DIF: Cognitive Level: Application/Applying or higher

1578 TOP: Nursing Process: Assessment MSC: NCLEX: Physiological Integrity 32. Which result for a 30-year-old patient with systemic lupus erythematosus (SLE) is most important for the nurse to communicate to the health care provider? a. Decreased C-reactive protein (CRP) b. Elevated blood urea nitrogen (BUN) c. Positive antinuclear antibodies (ANA) d. Positive lupus erythematosus cell prep

B The elevated BUN and creatinine levels indicate possible lupus nephritis and a need for a change in therapy to avoid further renal damage. The positive lupus erythematosus (LE) cell prep and ANA would be expected in a patient with SLE. A drop in CRP shows an improvement in the inflammatory process. DIF: Cognitive Level: Apply (application)

1575 TOP: Nursing Process: Planning MSC: NCLEX: Physiological Integrity 7. A patient with rheumatoid arthritis (RA) complains to the clinic nurse about having chronically dry eyes. Which action by the nurse is most appropriate? a. Teach the patient about adverse effects of the RA medications. b. Suggest that the patient use over-the-counter (OTC) artificial tears. c. Reassure the patient that dry eyes are a common problem with RA. d. Ask the health care provider about discontinuing methotrexate (Rheumatrex) .

B The patients dry eyes are consistent with Sjgrens syndrome, a common extraarticular manifestation of RA. Symptomatic therapy such as OTC eye drops is recommended. Dry eyes are not a side effect of methotrexate. Although dry eyes are common in RA, it is more helpful to offer a suggestion to relieve these symptoms than to offer reassurance. The dry eyes are not caused by RA treatment, but by the disease itself. DIF: Cognitive Level: Apply (application)

304 KEY: Hand-off communication| communication| The Joint Commission MSC: Integrated Process: Communication and Documentation NOT: Client Needs Category: Safe and Effective Care Environment: Management of Care 11. A nurse works in the rheumatology clinic and sees clients with rheumatoid arthritis (RA). Which client should the nurse see first? a. Client who reports jaw pain when eating b. Client with a red, hot, swollen right wrist c. Client who has a puffy-looking area behind the knee d. Client with a worse joint deformity since the last visit

B All of the options are possible manifestations of RA. However, the presence of one joint that is much redder, hotter, or more swollen that the other joints may indicate infection. The nurse needs to see this client first. DIF: Applying/Application

293 KEY: Osteoarthritis| acetaminophen| pharmacologic pain management| patient teaching MSC: Integrated Process: Teaching/Learning NOT: Client Needs Category: Physiological Integrity: Pharmacological and Parenteral Therapies 3. The clinic nurse assesses a client with diabetes during a checkup. The client also has osteoarthritis (OA). The nurse notes the clients blood glucose readings have been elevated. What question by the nurse is most appropriate? a. Are you compliant with following the diabetic diet? b. Have you been taking glucosamine supplements? c. How much exercise do you really get each week? d. Youre still taking your diabetic medication, right?

B All of the topics are appropriate for a client whose blood glucose readings have been higher than usual. However, since this client also has OA, and glucosamine can increase blood glucose levels, the nurse should ask about its use. The other questions all have an element of nontherapeutic communication in them. Compliant is a word associated with negative images, and the client may deny being noncompliant. Asking how much exercise the client really gets is accusatory. Asking if the client takes his or her medications right? is patronizing. DIF: Applying/Application

311 KEY: Rheumatoid arthritis| autoimmune disorders| nursing assessment| biologic response modifiers MSC: Integrated Process: Nursing Process: Assessment NOT: Client Needs Category: Safe and Effective Care Environment: Management of Care 17. A client with rheumatoid arthritis (RA) has an acutely swollen, red, and painful joint. What nonpharmacologic treatment does the nurse apply? a. Heating pad b. Ice packs c. Splints d. Wax dip

B Ice is best for acute inflammation. Heat often helps with joint stiffness. Splinting helps preserve joint function. A wax dip is used to provide warmth to the joint which is more appropriate for chronic pain and stiffness. DIF: Remembering/Knowledge

315 KEY: Systemic lupus erythematosus| autoimmune disorders| nursing assessment| pain| steroids MSC: Integrated Process: Nursing Process: Assessment NOT: Client Needs Category: Physiological Integrity: Pharmacological and Parenteral Therapies 20. A client with systemic lupus erythematosus (SLE) was recently discharged from the hospital after an acute exacerbation. The client is in the clinic for a follow-up visit and is distraught about the possibility of another hospitalization disrupting the family. What action by the nurse is best? a. Explain to the client that SLE is an unpredictable disease. b. Help the client create backup plans to minimize disruption. c. Offer to talk to the family and educate them about SLE. d. Tell the client to remain compliant with treatment plans.

B SLE is an unpredictable disease and acute exacerbations can occur without warning, creating chaos in the family. Helping the client make backup plans for this event not only will decrease the disruption but will give the client a sense of having more control. Explaining facts about the disease is helpful as well but does not engage the client in problem solving. The family may need education, but again this does not help the client to problem-solve. Remaining compliant may help decrease exacerbations, but is not as powerful an intervention as helping the client plan for such events. DIF: Applying/Application

316 KEY: Systemic lupus erythematosus| nursing evaluation| self-care| patient teaching| integumentary system MSC: Integrated Process: Nursing Process: Evaluation NOT: Client Needs Category: Health Promotion and Maintenance 29. A client is scheduled to have a hip replacement. Preoperatively, the client is found to be mildly anemic and the surgeon states the client may need a blood transfusion during or after the surgery. What action by the preoperative nurse is most important? a. Administer preoperative medications as prescribed. b. Ensure that a consent for transfusion is on the chart. c. Explain to the client how anemia affects healing. d. Teach the client about foods high in protein and iron.

B The preoperative nurse should ensure that all valid consents are on the chart, including one for blood transfusions if this may be needed. Administering preoperative medications is important for all preoperative clients and is not specific to this client. Teaching in the preoperative area should focus on immediate concerns. DIF: Applying/Application

296 KEY: Joint replacement| informed consent| blood transfusions| preoperative nursing MSC: Integrated Process: Communication and Documentation NOT: Client Needs Category: Safe and Effective Care Environment: Management of Care 30. An older client is scheduled to have hip replacement in 2 months and has the following laboratory values: white blood cell count: 8900/mm3, red blood cell count: 3.2/mm3, hemoglobin: 9 g/dL, hematocrit: 32%. What intervention by the nurse is most appropriate? a. Instruct the client to avoid large crowds. b. Prepare to administer epoetin alfa (Epogen). c. Teach the client about foods high in iron. d. Tell the client that all laboratory results are normal.

B This client is anemic, which needs correction prior to surgery. While eating iron-rich foods is helpful, to increase the clients red blood cells, hemoglobin, and hematocrit within 2 months, epoetin alfa is needed. This colony-stimulating factor will encourage the production of red cells. The clients white blood cell count is normal, so avoiding infection is not the priority. DIF: Applying/Application

293 KEY: Postoperative nursing| nonsteroidal anti-inflammatory drugs (NSAIDs)| musculoskeletal disorders MSC: Integrated Process: Communication and Documentation NOT: Client Needs Category: Physiological Integrity: Pharmacological and Parenteral Therapies MULTIPLE RESPONSE 1. The nursing student studying rheumatoid arthritis (RA) learns which facts about the disease? (Select all that apply.) a. It affects single joints only. b. Antibodies lead to inflammation. c. It consists of an autoimmune process. d. Morning stiffness is rare. e. Permanent damage is inevitable.

B, C RA is a chronic autoimmune systemic inflammatory disorder leading to arthritis-type symptoms in the joints and other symptoms that can be seen outside the joints. Antibodies are created that lead to inflammation. Clients often report morning stiffness. Permanent damage can be avoided with aggressive, early treatment. DIF: Remembering/Knowledge

305, 314, 317 KEY: Autoimmune disorders MSC: Integrated Process: Nursing Process: Assessment NOT: Client Needs Category: Physiological Integrity: Physiological Adaptation 7. A nurse works with several clients who have gout. Which types of gout and their drug treatments are correctly matched? (Select all that apply.) a. Allopurinol (Zyloprim) Acute gout b. Colchicine (Colcrys) Acute gout c. Febuxostat (Uloric) Chronic gout d. Indomethacin (Indocin) Acute gout e. Probenecid (Benemid) Chronic gout

B, C, D, E Acute gout can be treated with colchicine and indomethacin. Chronic gout can be treated with febuxostat and probenecid. Allopurinol is used for chronic gout. DIF: Remembering/Knowledge

N/A TOP: Client Needs Category: Psychosocial Integrity (Therapeutic Communication) MSC: Integrated Process: Caring MULTIPLE RESPONSE 1. What interventions does the nurse recommend for a client who is to be discharged home following total hip replacement surgery? (Select all that apply.) a. Continuous passive motion machine b. Elevated toilet seat c. Walker d. Crutches e. TED hose f. Heating pad

B, C, E The client will be using a walker, because crutches are used only by younger clients. TED hose should be worn until the client regains full mobility and Coumadin is discontinued. A walker will be needed until the client regains full strength and is able to walk with full weight bearing on the operative side. Crutches are not used because they do not provide enough support for the client during ambulation and pose a risk for falls. Heating pads increase blood flow to the area and may increase pain. Ice packs should be used instead, as needed. Continuous passive motion machines are not used after hip surgery. DIF: Cognitive Level: Application/Applying or higher

322 KEY: Fibromyalgia| patient education| physical modalities| nonpharmacologic pain management| complementary and alternative therapies MSC: Integrated Process: Teaching/Learning NOT: Client Needs Category: Health Promotion and Maintenance 4. The nurse working in the rheumatology clinic assesses clients with rheumatoid arthritis (RA) for late manifestations. Which signs/symptoms are considered late manifestations of RA? (Select all that apply.) a. Anorexia b. Feltys syndrome c. Joint deformity d. Low-grade fever e. Weight loss

B, C, E Late manifestations of RA include Feltys syndrome, joint deformity, weight loss, organ involvement, osteoporosis, extreme fatigue, and anemia, among others. Anorexia and low-grade fever are both seen early in the course of the disease. DIF: Remembering/Knowledge

N/A TOP: Client Needs Category: Physiological Integrity (Pharmacological and Parenteral TherapiesAdverse Effects/Contraindications/Interactions/Side Effects) MSC: Integrated Process: Nursing Process (Analysis) 3. The nurse is teaching a client with rheumatoid arthritis (RA) about joint protection principles. What information does the nurse include? (Select all that apply.) a. Use smaller joints to rest the larger ones. b. Hold objects with two hands, not one. c. Sit most often in a reclining chair. d. Use assistive-adaptive devices. e. Bend at your knees to lift objects.

B, D, E Clients with RA should use large joints to protect smaller ones, should hold objects with two hands instead of one, should sit in chairs with straight backs, should not bend at the waist but rather bend the knees while keeping the back straight, and should use assistive-adaptive devices wherever possible. DIF: Cognitive Level: Comprehension/Understanding

p. 327 TOP: Client Needs Category: Physiological Integrity (Pharmacological and Parenteral TherapiesExpected Actions/Outcomes) MSC: Integrated Process: Teaching/Learning 12. The nurse is caring for a client who had right total knee replacement surgery 3 days ago. During the assessment, the nurse notes that the clients right lower leg is twice the size of the left. What is the nurses priority intervention? a. Elevate the clients right leg. b. Apply antiembolism stockings. c. Assess the clients respiratory status. d. Check the clients pedal pulses.

C A common complication after total knee replacement (TKR) is the formation of a thrombus below the surgical site. This complication can lead to a pulmonary embolus and can be life threatening. Before notifying the surgeon or the emergency team, assess the clients pulmonary status to determine whether he or she has any manifestations of an embolus. The clients leg may be elevated and pedal pulses palpated, but respiratory assessment must be done first. TED hose should not be applied to a leg with suspected deep vein thrombosis (DVT). DIF: Cognitive Level: Application/Applying or higher

1592 TOP: Nursing Process: Evaluation MSC: NCLEX: Physiological Integrity 42. Which information will the nurse include when teaching a patient with newly diagnosed chronic fatigue syndrome about self-management? a. Avoid use of over-the-counter antihistamines or decongestants. b. A low-residue, low-fiber diet will reduce any abdominal distention. c. A gradual increase in your daily exercise may help decrease fatigue. d. Chronic fatigue syndrome usually progresses as patients become older.

C A graduated exercise program is recommended to avoid fatigue while encouraging ongoing activity. Because many patients with chronic fatigue syndrome have allergies, antihistamines and decongestants are used to treat allergy symptoms. A high-fiber diet is recommended. Chronic fatigue syndrome usually does not progress. DIF: Cognitive Level: Apply (application)

N/A TOP: Client Needs Category: Physiological Integrity (Reduction of Risk PotentialPotential for Complications From Surgical Procedures and Health Alterations) MSC: Integrated Process: Nursing Process (Evaluation) 5. A client returns to the medical-surgical unit after a total hip replacement with a large wedge-shaped pillow between his legs. The clients daughter asks the nurse why the pillow is in place. What is the nurses best response? a. It will help prevent bedsores from developing. b. It will help prevent nerve damage and foot drop. c. It will keep the new hip from becoming dislocated. d. It will prevent climbing out of bed if he becomes confused.

C Adduction of the operative leg beyond the midline could dislocate the new hip. The wedge pillow will help prevent this from happening. The wedge will not prevent bedsores from developing because it does not prevent pressure. The pillow will not prevent foot drop, because it is placed between the legs. The pillow is not a restraining device, and it will not prevent the client from climbing out of bed. DIF: Cognitive Level: Comprehension/Understanding

N/A TOP: Client Needs Category: Physiological Integrity (Reduction of Risk PotentialPotential for Alterations in Body Systems) MSC: Integrated Process: Teaching/Learning 22. The nurse has taught a client with lupus about skin protection in the clinic. Later, the nurse sees the client at an outdoor music festival. Which observation by the nurse indicates that the client requires further instruction? a. Client is wearing a thin, long-sleeved shirt. b. Client is wearing a hat with a full brim. c. Client is discussing her new perm. d. Client is seen applying sunscreen twice.

C Alopecia is common; the client should use gentle shampoo and avoid any harsh chemical treatments, such as a permanent wave. The other observations show good skin protection practices by the client. DIF: Cognitive Level: Comprehension/Understanding

1575 TOP: Nursing Process: Implementation MSC: NCLEX: Physiological Integrity 12. Anakinra (Kineret) is prescribed for a 49-year-old patient who has rheumatoid arthritis (RA). When teaching the patient about this drug, the nurse will include information about a. avoiding concurrently taking aspirin. b. symptoms of gastrointestinal (GI) bleeding. c. self-administration of subcutaneous injections. d. taking the medication with at least 8 oz of fluid.

C Anakinra is administered by subcutaneous injection. GI bleeding is not a side effect of this medication. Because the medication is injected, instructions to take it with 8 oz of fluid would not be appropriate. The patient is likely to be concurrently taking aspirin or nonsteroidal antiinflammatory drugs (NSAIDs), and these should not be discontinued. DIF: Cognitive Level: Apply (application)

1575 TOP: Nursing Process: Evaluation MSC: NCLEX: Physiological Integrity 31. A patient with an acute attack of gout in the right great toe has a new prescription for probenecid (Benemid). Which information about the patients home routine indicates a need for teaching regarding gout management? a. The patient sleeps about 8 to 10 hours every night. b. The patient usually eats beef once or twice a week. c. The patient takes one aspirin a day to prevent angina. d. The patient usually drinks about 3 quarts water daily.

C Aspirin interferes with the effectiveness of probenecid and should not be taken when the patient is taking probenecid. The patients sleep pattern will not affect gout management. Drinking 3 quarts of water and eating beef only once or twice a week are appropriate for the patient with gout. DIF: Cognitive Level: Apply (application)

1584 TOP: Nursing Process: Assessment MSC: NCLEX: Physiological Integrity 24. The nurse is planning care for a patient with hypertension and gout who has a red and painful right great toe. Which nursing action will be included in the plan of care? a. Gently palpate the toe to assess swelling. b. Use pillows to keep the right foot elevated. c. Use a footboard to hold bedding away from the toe. d. Teach patient to avoid use of acetaminophen (Tylenol).

C Because any touch on the area of inflammation may increase pain, bedding should be held away from the toe and touching the toe will be avoided. Elevation of the foot will not reduce the pain, which is caused by urate crystals. Acetaminophen can be used for pain relief. DIF: Cognitive Level: Understand (comprehension)

1573 TOP: Nursing Process: Assessment MSC: NCLEX: Physiological Integrity 35. Which laboratory data is important to communicate to the health care provider for a patient who is taking methotrexate (Rheumatrex) to treat rheumatoid arthritis? a. The blood glucose is 90 mg/dL. b. The rheumatoid factor is positive. c. The white blood cell (WBC) count is 1500/L. d. The erythrocyte sedimentation rate is elevated.

C Bone marrow suppression is a possible side effect of methotrexate, and the patients low WBC count places the patient at high risk for infection. The elevated erythrocyte sedimentation rate and positive rheumatoid factor are expected in rheumatoid arthritis. The blood glucose is normal. DIF: Cognitive Level: Apply (application)

1575 TOP: Nursing Process: Implementation MSC: NCLEX: Physiological Integrity 9. Which laboratory result will the nurse monitor to determine whether prednisone (Deltasone) has been effective for a 30-year-old patient with an acute exacerbation of rheumatoid arthritis? a. Blood glucose test b. Liver function tests c. C-reactive protein level d. Serum electrolyte levels

C C-reactive protein is a marker for inflammation, and a decrease would indicate that the corticosteroid therapy was effective. Blood glucose and serum electrolyte levels will also be monitored to check for side effects of prednisone. Liver function is not routinely monitored in patients receiving corticosteroids. DIF: Cognitive Level: Apply (application)

1564 TOP: Nursing Process: Evaluation MSC: NCLEX: Physiological Integrity 4. The nurse will anticipate the need to teach a 57-year-old patient who has osteoarthritis (OA) about which medication? a. Adalimumab (Humira) b. Prednisone (Deltasone) c. Capsaicin cream (Zostrix) d. Sulfasalazine (Azulfidine)

C Capsaicin cream blocks the transmission of pain impulses and is helpful for some patients in treating OA. The other medications would be used for patients with RA. DIF: Cognitive Level: Apply (application)

N/A TOP: Client Needs Category: Physiological Integrity (Pharmacological and Parenteral TherapiesExpected Actions/Outcomes) MSC: Integrated Process: Teaching/Learning 32. Which statement by a client indicates that additional teaching is needed in the management of fibromyalgia? a. I will switch to decaffeinated coffee in the mornings. b. Water aerobics classes will be a good form of exercise. c. Limiting my physical activity will reduce my fatigue. d. I will take my sertraline (Zoloft) right before I go to bed.

C Clients with fibromyalgia should be encouraged to exercise regularly, particularly performing activities that are low impact. Sleep disturbances are common in fibromyalgia, and anything that interferes with sleep, such as caffeine, should be avoided. Zoloft can cause drowsiness and should be taken daily at bedtime. DIF: Cognitive Level: Application/Applying or higher

p. 327 TOP: Client Needs Category: Physiological Integrity (Reduction of Risk PotentialPotential for Complications From Surgical Procedures and Health Alterations) MSC: Integrated Process: Teaching/Learning 10. The home care nurse is making a follow-up visit to a client who had total hip replacement surgery 2 weeks ago. Which client statement indicates a need for clarification regarding postoperative routine? a. My daughter helps me put on my elastic TED (thromboembolic deterrent) hose every day. b. I take 200 mg of Motrin (ibuprofen) at bedtime so that I can sleep. c. Now that my hip doesnt hurt, I can cross my legs like a lady again. d. Each day, I try to increase my walking time by at least 10 minutes.

C Crossing the legs beyond midline can dislocate the new hip joint and should be avoided at all times. The other statements demonstrate correct behavior and understanding. DIF: Cognitive Level: Application/Applying or higher

1588 TOP: Nursing Process: Evaluation MSC: NCLEX: Physiological Integrity 28. When the nurse brings medications to a patient with rheumatoid arthritis, the patient refuses the prescribed methotrexate (Rheumatrex). The patient tells the nurse, My arthritis isnt that bad yet. The side effects of methotrexate are worse than the arthritis. The most appropriate response by the nurse is a. You have the right to refuse to take the methotrexate. b. Methotrexate is less expensive than some of the newer drugs. c. It is important to start methotrexate early to decrease the extent of joint damage. d. Methotrexate is effective and has fewer side effects than some of the other drugs.

C Disease-modifying antirheumatic drugs (DMARDs) are prescribed early to prevent the joint degeneration that occurs as soon as the first year with RA. The other statements are accurate, but the most important point for the patient to understand is that it is important to start DMARDs as quickly as possible. DIF: Cognitive Level: Apply (application)

N/A TOP: Client Needs Category: Physiological Integrity (Pharmacological and Parenteral TherapiesAdverse Effects/Contraindications/Interactions/Side Effects) MSC: Integrated Process: Nursing Process (Assessment) 29. The nurse is caring for a pregnant client who is taking Humira (adalimumab) to control symptoms of rheumatoid arthritis. The client mentions the pain and inconvenience of the subcutaneous injections and asks, While Im pregnant, can I take this drug by mouth instead? What is the nurses best response? a. I will ask the physician to write a prescription for you today. b. Humira takes much longer to work when it is given orally. c. Humira can be given only by subcutaneous injection. d. You can switch from Humira to oral leflunomide (Arava).

C Humira is given by subcutaneous injection only. Arava causes birth defects; clients taking it must be on strict birth control and must inform their health care providers if pregnancy occurs. DIF: Cognitive Level: Application/Applying or higher

N/A TOP: Client Needs Category: Physiological Integrity (Physiological AdaptationPathophysiology) MSC: Integrated Process: Nursing Process (Assessment) 17. The nurse is teaching a client how to reduce the pain that she often experiences with fibromyalgia. Which statement does the nurse include in the teaching? a. Wear gloves outdoors in cooler temperatures. b. Avoid exercising when your muscles are sore. c. Make sure that you get enough sleep every night. d. Stay out of the sun as much as possible.

C In many clients, the pain of fibromyalgia occurs as a direct response to sleep deprivation. Encouraging the client to get sufficient sleep every night can drastically reduce the amount of pain experienced. Wearing gloves will not decrease the pain of fibromyalgia, but it may help a disease such as Raynauds phenomenon. Weight-bearing activities should not increase pain in a client with fibromyalgia. Similarly, sun exposure has not been identified as a causative pain factor. DIF: Cognitive Level: Application/Applying or higher

N/A TOP: Client Needs Category: Health Promotion and Maintenance (Self-Care) MSC: Integrated Process: Teaching/Learning 20. The school nurse is working with a group of high school students who will be going on a field trip to a nature center. Which student is at highest risk for a tick bite? a. Male student with a beard and a baseball cap b. Female student with long hair pulled back in a ponytail c. Male student wearing a long-sleeved shirt and shorts d. Female student who is wearing scented hand lotion

C Long pants should be worn and tucked into socks or boots to help prevent tick bites. Facial hair, hats, ponytails, and scented body products do not increase the risk for tick bites. DIF: Cognitive Level: Comprehension/Understanding

1581 TOP: Nursing Process: Implementation MSC: NCLEX: Physiological Integrity 15. A 19-year-old patient hospitalized with a fever and red, hot, and painful knees is suspected of having septic arthritis. Information obtained during the nursing history that indicates a risk factor for septic arthritis is that the patient a. had several knee injuries as a teenager. b. recently returned from South America. c. is sexually active with multiple partners. d. has a parent who has rheumatoid arthritis.

C Neisseria gonorrhoeae is the most common cause for septic arthritis in sexually active young adults. The other information does not point to any risk for septic arthritis. DIF: Cognitive Level: Understand (comprehension)

N/A TOP: Client Needs Category: Health Promotion and Maintenance (Principles of Teaching/Learning) MSC: Integrated Process: Teaching/Learning 19. The nurse provides discharge teaching for a client to prevent a new attack of gout. Which statement by the client indicates that additional teaching is required? a. I will keep a food and symptom diary for a few weeks. b. If I get a headache, I will take Tylenol instead of aspirin. c. I hate to start limiting my fluid intake so much! d. Citrus juices and milk may keep me from having kidney stones.

C Nutritional therapy for gout is controversial; however, clients do need to increase their fluid intake to prevent kidney stones. Certain foods may precipitate an acute attack, so clients should learn to determine which foods trigger their gout. Aspirin is well known to trigger gout attacks. Increasing the intake of alkaline ash foods such as citrus juices and milk might prevent the formation of kidney stones. DIF: Cognitive Level: Application/Applying or higher

1566 OBJ: Special Questions: Prioritization TOP: Nursing Process: Planning MSC: NCLEX: Physiological Integrity 39. Which patient seen by the nurse in the outpatient clinic is most likely to require teaching about ways to reduce risk for osteoarthritis (OA)? a. A 38-year-old man who plays on a summer softball team b. A 56-year-old man who is a member of a construction crew c. A 56-year-old woman who works on an automotive assembly line d. A 49-year-old woman who is newly diagnosed with diabetes mellitus

C OA is more likely to occur in women as a result of estrogen reduction at menopause and in individuals whose work involves repetitive movements and lifting. Moderate exercise, such as softball, reduces risk for OA. Diabetes is not a risk factor for OA. Working on a construction crew would involve nonrepetitive work and thus would not be as risky. DIF: Cognitive Level: Apply (application)

p. 346 TOP: Client Needs Category: Health Promotion and Maintenance (Self-Care) MSC: Integrated Process: Nursing Process (Assessment) 23. A client with diagnosed osteoarthritis comes to the clinic reporting a low-grade fever, fatigue, and bilateral joint pain. What action by the nurse is most appropriate? a. Assess the client for a systemic infection. b. Discuss increasing the dose of anti-arthritis drugs. c. Prepare the client for a laboratory draw for rheumatoid factor. d. Teach the client joint protection activities.

C Osteoarthritis is generally a unilateral disease. The manifestations that this client exhibits are more consistent with rheumatoid arthritis, so the nurse will prepare the client for a blood draw. The nurse may need to teach joint protection measures, but an accurate diagnosis is most important. DIF: Cognitive Level: Application/Applying or higher

1578 TOP: Nursing Process: Assessment MSC: NCLEX: Physiological Integrity 17. A 29-year-old patient reporting painful urination and knee pain is diagnosed with reactive arthritis. The nurse will plan to teach the patient about the need for several months of therapy with a. anakinra (Kineret). b. etanercept (Enbrel). c. doxycycline (Vibramycin). d. methotrexate (Rheumatrex).

C Reactive arthritis associated with urethritis is usually caused by infection with Chlamydia trachomatis and requires 3 months of treatment with doxycycline. The other medications are used for chronic inflammatory problems such as rheumatoid arthritis. DIF: Cognitive Level: Apply (application)

1564 TOP: Nursing Process: Planning MSC: NCLEX: Physiological Integrity 5. A patient with rheumatoid arthritis being seen in the clinic has rheumatoid nodules on the elbows. Which action will the nurse take? a. Draw blood for rheumatoid factor analysis. b. Teach the patient about injections for the nodules. c. Assess the nodules for skin breakdown or infection. d. Discuss the need for surgical removal of the nodules.

C Rheumatoid nodules can break down or become infected. They are not associated with changes in rheumatoid factor, and injection is not needed. Rheumatoid nodules are usually not removed surgically because of a high probability of recurrence. DIF: Cognitive Level: Apply (application)

1586 TOP: Nursing Process: Diagnosis MSC: NCLEX: Psychosocial Integrity 23. A new clinic patient with joint swelling and pain is being tested for systemic lupus erythematosus. Which test will provide the most specific findings for the nurse to review? a. Rheumatoid factor (RF) b. Antinuclear antibody (ANA) c. Anti-Smith antibody (Anti-Sm) d. Lupus erythematosus (LE) cell prep

C The anti-Sm is antibody found almost exclusively in SLE. The other blood tests are also used in screening but are not as specific to SLE. DIF: Cognitive Level: Apply (application)

N/A TOP: Client Needs Category: Physiological Integrity (Pharmacological and Parenteral TherapiesPharmacological Pain Management) MSC: Integrated Process: Nursing Process (Assessment) 44. The nurse is caring for a client who has a history of severe rheumatoid arthritis. The client becomes combative and abusive to the staff when she is unable to perform personal care independently. What is the best statement the nurse can make to the client at this time? a. I will have to restrain your hands if you cannot keep them to yourself. b. I will ask your doctor for a psychiatrist to talk to you about anger management. c. You seem frustrated. Would you like to try to dress again in a few minutes? d. Would you like me to get an order for medication to help you settle down?

C The client is acting out her frustration over her chronic illness and loss of use of her hands. The nurse should acknowledge this frustration. Allowing the client to make decisions regarding care will help the client regain some sense of control and will help improve self-esteem. Requesting sedation, suggesting psychiatric therapy, or threatening use of restraints is not appropriate, because the client is expressing frustration over the situation. DIF: Cognitive Level: Application/Applying or higher

313 KEY: Rheumatoid arthritis| autoimmune disorders| coping| culture| patient-centered care| diversity MSC: Integrated Process: Caring NOT: Client Needs Category: Psychosocial Integrity 34. A client has rheumatoid arthritis that especially affects the hands. The client wants to finish quilting a baby blanket before the birth of her grandchild. What response by the nurse is best? a. Lets ask the provider about increasing your pain pills. b. Hold ice bags against your hands before quilting. c. Try a paraffin wax dip 20 minutes before you quilt. d. You need to stop quilting before it destroys your fingers.

C Paraffin wax dips are beneficial for decreasing pain in arthritic hands and lead to increased mobility. The nurse can suggest this comfort measure. Increasing pain pills will not help with movement. Ice has limited use unless the client has a hot or exacerbated joint. The client wants to finish her project, so the nurse should not negate its importance by telling the client it is destroying her joints. DIF: Applying/Application

N/A TOP: Client Needs Category: Health Promotion and Maintenance (Self-Care) MSC: Integrated Process: Nursing Process (Assessment) 42. The nurse is caring for an older adult client who has fallen and fractured her hip. The client will have hip replacement surgery followed by extensive rehabilitation. The client confides in the nurse, I feel like I dont have any control over anything anymore now that I am old. What is the nurses best response? a. Ill make sure that the physical and occupational therapists see you after surgery to help get your strength back. b. Its normal to feel this way, but hopefully you will be back on your feet after a stay in rehab. c. Its important to control what you can right now, like making out your menu every day and working with the therapists. d. I sense that you are feeling depressed about the situation. I will ask the doctor to prescribe an antidepressant for you.

C The nurse should support the clients self-esteem and increase feelings of competency by encouraging activities that assist in maintaining some degree of control, such as participation in decision making and performance of tasks that he or she can manage. The nurse should provide immediate control options for the client, rather than waiting until after rehabilitation. The clients desire for control does not indicate depression, so an antidepressant is not indicated. Therapy referrals are appropriate but do not address the clients desire for control. DIF: Cognitive Level: Application/Applying or higher

1579 TOP: Nursing Process: Assessment MSC: NCLEX: Physiological Integrity 16. The nurse notices a circular lesion with a red border and clear center on the arm of an 18-year-old summer camp counselor who is in the camp clinic complaining of chills and muscle aches. Which action should the nurse takenext? a. Palpate the abdomen. b. Auscultate the heart sounds. c. Ask the patient about recent outdoor activities. d. Question the patient about immunization history.

C The patients clinical manifestations suggest possible Lyme disease. A history of recent outdoor activities such as hikes will help confirm the diagnosis. The patients symptoms do not suggest cardiac or abdominal problems or lack of immunization. DIF: Cognitive Level: Apply (application)

297 KEY: Nursing assessment| joint replacement| musculoskeletal system MSC: Integrated Process: Nursing Process: Assessment NOT: Client Needs Category: Physiological Integrity: Reduction of Risk Potential 8. A client has a continuous passive motion (CPM) device after a total knee replacement. What action does the nurse delegate to the unlicensed assistive personnel (UAP) after the affected leg is placed in the machine while the client is in bed? a. Assess the distal circulation in 30 minutes. b. Change the settings based on range of motion. c. Raise the lower siderail on the affected side. d. Remind the client to do quad-setting exercises.

C Because the clients leg is strapped into the CPM, if it falls off the bed due to movement, the clients leg (and new joint) can be injured. The nurse should instruct the UAP to raise the siderail to prevent this from occurring. Assessment is a nursing responsibility. Only the surgeon, physical therapist, or specially trained technician adjusts the CPM settings. Quad-setting exercises are not related to the CPM machine. DIF: Applying/Application

299 KEY: Joint replacement| safety| falls| musculoskeletal system MSC: Integrated Process: Nursing Process: Implementation NOT: Client Needs Category: Safe and Effective Care Environment: Safety and Infection Control 32. A client has fibromyalgia and is prescribed duloxetine hydrochloride (Cymbalta). The client calls the clinic and asks the nurse why an antidepressant drug has been prescribed. What response by the nurse is best? a. A little sedation will help you get some rest. b. Depression often accompanies fibromyalgia. c. This drug works in the brain to decrease pain. d. You will have more energy after taking this drug.

C Duloxetine works to increase the release of the neurotransmitters serotonin and norepinephrine, which reduces the pain from fibromyalgia. The other answers are inaccurate. DIF: Understanding/Comprehension

311 KEY: Rheumatoid arthritis| autoimmune disorders| nonpharmacologic pain management| heat MSC: Integrated Process: Caring NOT: Client Needs Category: Physiological Integrity: Basic Care and Comfort 35. A client has newly diagnosed systemic lupus erythematosus (SLE). What instruction by the nurse is most important? a. Be sure you get enough sleep at night. b. Eat plenty of high-protein, high-iron foods. c. Notify your provider at once if you get a fever. d. Weigh yourself every day on the same scale.

C Fever is the classic sign of a lupus flare and should be reported immediately. Rest and nutrition are important but do not take priority over teaching the client what to do if he or she develops an elevated temperature. Daily weights may or may not be important depending on renal involvement. DIF: Understanding/Comprehension

Chart 20-6, p. 332 TOP: Client Needs Category: Health Promotion and Maintenance (Self-Care) MSC: Integrated Process: Teachi 1. A nurse is working with a community group promoting healthy aging. What recommendation is best to help prevent osteoarthritis (OA)? a. Avoid contact sports. b. Get plenty of calcium. c. Lose weight if needed. d. Engage in weight-bearing exercise.

C Obesity can lead to OA, and if the client is overweight, losing weight can help prevent OA or reduce symptoms once it occurs. Arthritis can be caused by contact sports, but this is less common than obesity. Calcium and weight-bearing exercise are both important for osteoporosis. DIF: Understanding/Comprehension

295 KEY: Osteoarthritis| nursing assessment| supplements MSC: Integrated Process: Nursing Process: Assessment NOT: Client Needs Category: Physiological Integrity: Pharmacological and Parenteral Therapies 4. The nurse working in the orthopedic clinic knows that a client with which factor has an absolute contraindication for having a total joint replacement? a. Needs multiple dental fillings b. Over age 85 c. Severe osteoporosis d. Urinary tract infection

C Osteoporosis is a contraindication to joint replacement because the bones have a high risk of shattering as the new prosthesis is implanted. The client who needs fillings should have them done prior to the surgery. Age greater than 85 is not an absolute contraindication. A urinary tract infection can be treated prior to surgery. DIF: Remembering/Knowledge

297 KEY: Joint replacement| infection control| wound infection| dressings MSC: Integrated Process: Nursing Process: Implementation NOT: Client Needs Category: Safe and Effective Care Environment: Safety and Infection Control 26. A nurse is discharging a client after a total hip replacement. What statement by the client indicates good potential for self-management? a. I can bend down to pick something up. b. I no longer need to do my exercises. c. I will not sit with my legs crossed. d. I wont wash my incision to keep it dry.

C There are many precautions clients need to take after hip replacement surgery, including not bending more than 90 degrees at the hips, continuing prescribed exercises, not crossing the legs, and washing the incision daily and patting it dry. DIF: Evaluating/Synthesis

301 KEY: Joint replacement| delegation| continuous passive motion machine| unlicensed assistive personnel (UAP) MSC: Integrated Process: Nursing Process: Implementation NOT: Client Needs Category: Safe and Effective Care Environment: Management of Care 9. After a total knee replacement, a client is on the postoperative nursing unit with a continuous femoral nerve blockade. On assessment, the nurse notes the clients pulses are 2+/4+ bilaterally; the skin is pale pink, warm, and dry; and the client is unable to dorsiflex or plantarflex the affected foot. What action does the nurse perform next? a. Document the findings and monitor as prescribed. b. Increase the frequency of monitoring the client. c. Notify the surgeon or anesthesia provider immediately. d. Palpate the clients bladder or perform a bladder scan.

C With the femoral nerve block, the client should still be able to dorsiflex and plantarflex the affected foot. Since this client has an abnormal finding, the nurse should notify either the surgeon or the anesthesia provider immediately. Documentation is the last priority. Increasing the frequency of assessment may be a good idea, but first the nurse must notify the appropriate person. Palpating the bladder is not related. DIF: Applying/Application

320 KEY: Gout| pain| pharmacologic pain management MSC: Integrated Process: Teaching/Learning NOT: Client Needs Category: Physiological Integrity: Pharmacological and Parenteral Therapies 8. The nurse is caring for a client with systemic sclerosis (SSc). What comfort measures can the nurse delegate to the unlicensed assistive personnel (UAP)? (Select all that apply.) a. Collaborate with a registered dietitian for appropriate foods. b. Inspect the skin and note any areas of ulceration. c. Keep the room at a comfortably warm temperature. d. Place a foot cradle at the end of the bed to lift sheets. e. Remind the client to elevate the head of the bed after eating.

C, D, E The client with SSc should avoid cold temperatures, which may lead to vasospasms and Raynauds phenomenon. The UAP can adjust the room temperature for the clients comfort. Keeping the sheets off the feet will help prevent injury; the UAP can apply a foot cradle to the bed to hold the sheets up. Because of esophageal problems, the client should remain in an upright position for 1 to 2 hours after meals. The UAP can remind the client of this once he or she has been taught. The other actions are performed by the registered nurse. DIF: Applying/Application

N/A TOP: Client Needs Category: Physiological Integrity (Reduction of Risk PotentialPotential for Complications from Surgical Procedures and Health Alterations) MSC: Integrated Process: Nursing Process (Analysis) 38. A nurse is caring for a client who has had rheumatoid arthritis (RA) for 5 years. Which laboratory value requires the most immediate intervention by the nurse? a. White blood cell count (WBC), 3800/mm3 b. Hemoglobin (Hg), 10.6 g/dL c. Blood urea nitrogen (BUN), 16 mg/dL d. Creatinine, 3.2 mg/dL

D Clients with RA usually have pancytopenia, or a decrease in all cell types. WBC and hemoglobin are low, consistent with this condition. BUN is normal. Creatinine is very high; this indicates renal disease. This client may have renal consequences of his or her RA, which should be investigated. DIF: Cognitive Level: Application/Applying or higher

1578 TOP: Nursing Process: Planning MSC: NCLEX: Physiological Integrity 20. A 71-year-old patient who takes multiple medications develops acute gouty arthritis. The nurse will consult with the health care provider before giving the prescribed dose of a. sertraline (Zoloft). b. famotidine (Pepcid). c. oxycodone (Roxicodone). d. hydrochlorothiazide (HydroDIURIL).

D Diuretic use increases uric acid levels and can precipitate gout attacks. The other medications are safe to administer. DIF: Cognitive Level: Apply (application)

N/A TOP: Client Needs Category: Physiological Integrity (Basic Care and ComfortNon-Pharmacological Comfort Interventions) MSC: Integrated Process: Nursing Process (Implementation) 16. A client is suspected to have rheumatoid arthritis. Which manifestations does the nurse assess this client carefully for? a. Crepitus when the client moves the shoulders b. Numbness and tingling in the clients fingers c. Client has cool feet, with weak pedal pulses d. Low-grade fever, fatigue, anorexia with weight loss

D Low-grade fever is common with rheumatoid arthritis because of the inflammatory response. Fatigue, anorexia, and weight loss are also common symptoms. Impaired neurologic status, popping sounds with range of motion (ROM), and poor circulation are not common symptoms of rheumatoid arthritis. DIF: Cognitive Level: Application/Applying or higher

1568 TOP: Nursing Process: Planning MSC: NCLEX: Physiological Integrity 43. After the nurse assesses a 78-year-old who uses naproxen (Aleve) daily for hand and knee osteoarthritis management, which information is most important to report to the health care provider? a. Knee crepitation is noted with normal knee range of motion. b. Patient reports embarrassment about having Heberdens nodes. c. Patients knee pain while golfing has increased over the last year. d. Laboratory results indicate blood urea nitrogen (BUN) is elevated.

D Older patients are at increased risk for renal toxicity caused by nonsteroidal antiinflammatory drugs (NSAIDs) such as naproxen. The other information will also be reported to the health care provider but is consistent with the patients diagnosis of osteoarthritis and will not require an immediate change in the patients treatment plan. DIF: Cognitive Level: Apply (application)

1571 TOP: Nursing Process: Evaluation MSC: NCLEX: Physiological Integrity 10. The nurse teaching a support group of women with rheumatoid arthritis (RA) about how to manage activities of daily living suggests that they a. stand rather than sit when performing household and yard chores. b. strengthen small hand muscles by wringing sponges or washcloths. c. protect the knee joints by sleeping with a small pillow under the knees. d. avoid activities that require repetitive use of the same muscles and joints.

D Patients are advised to avoid repetitious movements. Sitting during household chores is recommended to decrease stress on joints. Wringing water out of sponges would increase the joint stress. Patients are encouraged to position joints in the extended position, and sleeping with a pillow behind the knees would decrease the ability of the knee to extend and also decrease knee range of motion (ROM). DIF: Cognitive Level: Apply (application)

N/A TOP: Client Needs Category: Physiological Integrity (Pharmacological and Parenteral TherapiesAdverse Effects/Contraindications/Interactions/Side Effects) MSC: Integrated Process: Nursing Process (Implementation) 31. The nurse is instructing a client about management of discoid lupus erythematosus (DLE). Which statement indicates that the client requires additional teaching? a. I will be sure to apply sunscreen whenever I am outside. b. I will apply small amounts of the steroid cream to my face twice a day. c. I will take Plaquenil (hydroxychloroquine sulfate) with breakfast each morning. d. Steroids weaken the immune system, so I will wash my hands frequently.

D Steroid creams used for the treatment of discoid lupus will not weaken the immune system because they should be applied in small amounts to affected areas. The client will be more sensitive to sun exposure while using the steroid cream, so sunscreen should be used whenever the client goes outside. The client should use only small amounts of the cream on her face. Plaquenil should be taken with meals or a glass of milk. DIF: Cognitive Level: Application/Applying or higher

15 OBJ: Special Questions: Delegation TOP: Nursing Process: Planning MSC: NCLEX: Physiological Integrity 46. When reviewing the chart shown in the accompanying figure for a new patient with rheumatoid arthritis, the nurse reads that the patient has swan neck deformities. Which deformity will the nurse expect to observe when assessing the patient? a. A b. B c. C d. D

D Swan neck deformity involves distal interphalangeal joint hyperflexion and proximal interphalangeal joint hyperextension of the hands. The other deformities are also associated with rheumatoid arthritis: ulnar drift, boutonniere deformity, and hallux vagus. DIF: Cognitive Level: Understand (comprehension)

N/A TOP: Client Needs Category: Health Promotion and Maintenance (Self-Care) MSC: Integrated Process: Teaching/Learning 2. The nurse is teaching a client who has osteoarthritis ways to slow progression of the disease. Which statement indicates that the client understands the nurses instruction? a. I will eat more vegetables and less meat. b. I will avoid exercising to minimize wear on my joints. c. I will take calcium with vitamin D every day. d. I will start swimming twice a week.

D Swimming is an excellent form of exercise for clients with arthritis because it involves minimal weight bearing and stress on the joints from gravity. Eating more vegetables will not decrease the progression of osteoarthritis. Taking calcium with vitamin D will decrease the risk of osteoporosis, not osteoarthritis. Gentle exercise is important to help slow progression of the disease. DIF: Cognitive Level: Application/Applying or higher

1566 TOP: Nursing Process: Evaluation MSC: NCLEX: Physiological Integrity 30. The home health nurse is doing a follow-up visit to a 41-year-old patient with recently diagnosed rheumatoid arthritis (RA). Which assessment made by the nurse indicates that more patient teaching is needed? a. The patient takes a 2-hour nap each day. b. The patient has been taking 16 aspirins daily. c. The patient sits on a stool while preparing meals. d. The patient sleeps with two pillows under the head.

D The joints should be maintained in an extended position to avoid contractures, so patients should use a small, flat pillow for sleeping. The other information is appropriate for a patient with RA and indicates that teaching has been effective. DIF: Cognitive Level: Apply (application)

1586 TOP: Nursing Process: Evaluation MSC: NCLEX: Physiological Integrity 22. A 25-year-old female patient with systemic lupus erythematosus (SLE) who has a facial rash and alopecia tells the nurse, I never leave my house because I hate the way I look. An appropriate nursing diagnosis for the patient is a. activity intolerance related to fatigue and inactivity. b. impaired social interaction related to lack of social skills. c. impaired skin integrity related to itching and skin sloughing. d. social isolation related to embarrassment about the effects of SLE.

D The patients statement about not going anywhere because of hating the way he or she looks supports the diagnosis of social isolation because of embarrassment about the effects of the SLE. Activity intolerance is a possible problem for patients with SLE, but the information about this patient does not support this as a diagnosis. The rash with SLE is nonpruritic. There is no evidence of lack of social skills for this patient. DIF: Cognitive Level: Apply (application)

N/A TOP: Client Needs Category: Physiological Integrity (Reduction of Risk PotentialLaboratory Values) MSC: Integrated Process: Nursing Process (Analysis) 39. The nurse is caring for an older adult client who will be discharged after being hospitalized for a total hip replacement. Which statement indicates that arrangements may have to be made to have the clients medications supervised at home? a. I will take my Coumadin pill every day just before the evening news. b. My wife takes iron too, so we will take our pills together every morning. c. I prepare all my pills for the week and will place them in a labeled medi-set. d. If my legs get swollen, I will take an extra Coumadin pill that day.

D Warfarin (Coumadin) is an anticoagulant prescribed to prevent venous thromboembolism after joint replacement surgery. It is not used for edema. The other statements show that the client has an appropriate plan for self-administration of his medications. DIF: Cognitive Level: Application/Applying or higher

302 KEY: Postoperative nursing| joint replacement| nursing assessment| musculoskeletal system MSC: Integrated Process: Nursing Process: Assessment NOT: Client Needs Category: Physiological Integrity: Reduction of Risk Potential 10. A nurse is discharging a client to a short-term rehabilitation center after a joint replacement. Which action by the nurse is most important? a. Administering pain medication before transport b. Answering any last-minute questions by the client c. Ensuring the family has directions to the facility d. Providing a verbal hand-off report to the facility

D As required by The Joint Commission and other accrediting agencies, a hand-off report must be given to the new provider to prevent error. The other options are valid responses but do not take priority. DIF: Applying/Application

305 KEY: Rheumatoid arthritis| nursing assessment| autoimmune disorder MSC: Integrated Process: Nursing Process: Assessment NOT: Client Needs Category: Safe and Effective Care Environment: Management of Care 12. A client with rheumatoid arthritis (RA) is on the postoperative nursing unit after having elective surgery. The client reports that one arm feels like pins and needles and that the neck is very painful since returning from surgery. What action by the nurse is best? a. Assist the client to change positions. b. Document the findings in the clients chart. c. Encourage range of motion of the neck. d. Notify the provider immediately.

D Clients with RA can have cervical joint involvement. This can lead to an emergent situation in which the phrenic nerve is compressed, causing respiratory insufficiency. The client can also suffer a permanent spinal cord injury. The nurse needs to notify the provider immediately. Changing positions and doing range of motion may actually worsen the situation. The nurse should document findings after notifying the provider. DIF: Applying/Application

298 KEY: Joint replacement| discharge planning/teaching| nursing evaluation MSC: Integrated Process: Nursing Process: Evaluation NOT: Client Needs Category: Health Promotion and Maintenance 27. The nurse is caring for a client using a continuous passive motion (CPM) machine and has delegated some tasks to the unlicensed assistive personnel (UAP). What action by the UAP warrants intervention by the nurse? a. Checking to see if the machine is working b. Keeping controls in a secure place on the bed c. Placing padding in the machine per request d. Storing the CPM machine under the bed after removal

D For infection control (and to avoid tripping on it), the CPM machine is never placed on the floor. The other actions are appropriate. DIF: Applying/Application

295 KEY: Osteoarthritis| osteoporosis| joint replacement| surgical procedures MSC: Integrated Process: Nursing Process: Assessment NOT: Client Needs Category: Physiological Integrity: Reduction of Risk Potential 5. An older client has returned to the surgical unit after a total hip replacement. The client is confused and restless. What intervention by the nurse is most important to prevent injury? a. Administer mild sedation. b. Keep all four siderails up. c. Restrain the clients hands. d. Use an abduction pillow.

D Older clients often have trouble metabolizing anesthetics and pain medication, leading to confusion or restlessness postoperatively. To prevent the hip from dislocating, the nurse should use an abduction pillow since the client cannot follow directions at this time. Sedation may worsen the clients mental status and should be avoided. Using all four siderails may be considered a restraint. Hand restraints are not necessary in this situation. DIF: Applying/Application

306 KEY: Rheumatoid arthritis| autoimmune disorders| nursing assessment MSC: Integrated Process: Nursing Process: Assessment NOT: Client Needs Category: Physiological Integrity: Reduction of Risk Potential 25. A nurse is caring for a client after joint replacement surgery. What action by the nurse is most important to prevent wound infection? a. Assess the clients white blood cell count. b. Culture any drainage from the wound. c. Monitor the clients temperature every 4 hours. d. Use aseptic technique for dressing changes.

D Preventing surgical wound infection is a primary responsibility of the nurse, who must use aseptic technique to change dressings or empty drains. The other actions do not prevent infection but can lead to early detection of an infection that is already present. DIF: Applying/Application

306 KEY: Rheumatoid arthritis| autoimmune disorder| musculoskeletal system| communication| critical rescue MSC: Integrated Process: Nursing Process: Implementation NOT: Client Needs Category: Physiological Integrity: Physiological Adaptation 13. The nurse working in the rheumatology clinic is seeing clients with rheumatoid arthritis (RA). What assessment would be most important for the client whose chart contains the diagnosis of Sjgrens syndrome? a. Abdominal assessment b. Oxygen saturation c. Renal function studies d. Visual acuity

D Sjgrens syndrome is seen in clients with RA and manifests with dryness of the eyes, mouth, and vagina in females. Visual disturbances can occur. The other assessments are not related to RA and Sjgrens syndrome. DIF: Applying/Application

310 KEY: Rheumatoid arthritis| autoimmune disease| biologic response modifiers| client education MSC: Integrated Process: Teaching/Learning NOT: Client Needs Category: Health Promotion and Maintenance 16. The nurse in the rheumatology clinic is assessing clients with rheumatoid arthritis (RA). Which client should the nurse see first? a. Client taking celecoxib (Celebrex) and ranitidine (Zantac) b. Client taking etanercept (Enbrel) with a red injection site c. Client with a blood glucose of 190 mg/dL who is taking steroids d. Client with a fever and cough who is taking tofacitinib (Xeljanz)

D Tofacitinib carries a Food and Drug Administration black box warning about opportunistic infections, tuberculosis, and cancer. Fever and cough may indicate tuberculosis. Ranitidine is often taken with celecoxib, which can cause gastrointestinal distress. Redness and itchy rashes are frequently seen with etanercept injections. Steroids are known to raise blood glucose levels. DIF: Applying/Application

322 KEY: Fibromyalgia| rest and sleep| patient-centered care MSC: Integrated Process: Nursing Process: Implementation NOT: Client Needs Category: Physiological Integrity: Basic Care and Comfort 12. A client has a possible connective tissue disease and the nurse is reviewing the clients laboratory values. Which laboratory values and their related connective tissue diseases (CTDs) are correctly matched? (Select all that apply.) a. Elevated antinuclear antibody (ANA) Normal value; no connective tissue disease b. Elevated sedimentation rate Rheumatoid arthritis c. Lowered albumin Indicative only of nutritional deficit d. Positive human leukocyte antigen B27 (HLA-B27) Reiters syndrome or ankylosing spondylitis e. Positive rheumatoid factor Possible kidney disease

D, E The HLA-B27 is diagnostic for Reiters syndrome or ankylosing spondylitis. A positive rheumatoid factor can be seen in autoimmune CTDs, kidney and liver disease, or leukemia. An elevated ANA is indicative of inflammatory CTDs, although a small minority of healthy adults also have this finding. An elevated sedimentation rate indicates inflammation, whether from an infection, an injury, or an autoimmune CTD. Lowered albumin is seen in nutritional deficiencies but also in chronic infection or inflammation. DIF: Remembering/Knowledge


Kaugnay na mga set ng pag-aaral

Chapter 61: Management of Patients with Dermatologic Disorders

View Set

Ch. 13 Med-Surg: Palliative and End-of-Life Care

View Set

Quiz Results - SBE Refresher 1 - Defining Research with Human Subjects

View Set

RN Fundamentals Online Practice 2019 A with NGN

View Set

energy and the law of conservation of energy

View Set

MCB W61 Final QUIZ QUESTIONS, MCB C61 Final Quizzy, PSYCH / MCB C61 Final, BMB Test 3, MCB C61 Midterm 2, MCB c61 Midterm 2, Psych C 61 midterm 2

View Set